CSE-PRELIMS 2018 • PAPER-1 · 2 | Detailed of Solutions of CSE-Prelims:2018 • Paper-1 Analysis...

47
BIG LEARNINGS MADE EASY www.nextias.com Detailed Solutions Saket Centre: 316/274, Westend Marg, Saidulajab, Near Saket Metro Station, New Delhi-110030 Old Rajinder Nagar Centre: Ground Floor, 6, Old Rajinder Nagar (Near Salwan School Gate No. 2), New Delhi 110060 Ph: 8800338066 | S CSE-PRELIM 2018 • PAPER-1

Transcript of CSE-PRELIMS 2018 • PAPER-1 · 2 | Detailed of Solutions of CSE-Prelims:2018 • Paper-1 Analysis...

Page 1: CSE-PRELIMS 2018 • PAPER-1 · 2 | Detailed of Solutions of CSE-Prelims:2018 • Paper-1 Analysis of 2018 Prelims: 1. The question paper was difficult as compared to the last year’s

An initiative of Group

B I G L E A R N I N G S M A D E E ASY

www.nextias.com

Detailed Solutions

Saket Centre: 316/274, Westend Marg, Saidulajab, Near Saket Metro Station, New Delhi-110030

Old Rajinder Nagar Centre: Ground Floor, 6, Old Rajinder Nagar (Near Salwan School Gate No. 2), New Delhi 110060

Ph: 8800338066 |

S CSE-PRELIM 2018 • PAPER-1

Page 2: CSE-PRELIMS 2018 • PAPER-1 · 2 | Detailed of Solutions of CSE-Prelims:2018 • Paper-1 Analysis of 2018 Prelims: 1. The question paper was difficult as compared to the last year’s

2 | Detailed of Solutions of CSE-Prelims:2018 • Paper-1

Analysis of 2018 Prelims:

1. The question paper was difficult as compared to the last year’s Prelims paper.

2. There were many questions where options were set in such a way that it was not easy to solve the question using elimination.

3. There was focus on questions from the Current Affairs and issues in the news. But a considerable number of questions were asked based on events which were in news 2-3 years ago.

4. The questions from conventional topics like Modern Indian History, Science and Technology, Indian Economy etc required in-depth knowledge of the topics.

5. This year, around 22 questions were asked from History section (including Ancient, Medieval, Modern History, Post-Independence and Art & Culture).

6. In last three years UPSC has started asking more questions from Modern India Personalities, their work and organizations.

7. The share of questions based on environment and ecology has reduced in last 2 years. Moreover, in the environment and ecology section questions mainly focused on the environment organizations, treaties and environmental degradation.

8. Flagship schemes of government were asked with their detailed provisions like RTE, PMKVY etc. Moreover, the details asked in these questions are generally not covered in regular newspapers.

Suggestions based on current paper

1. As UPSC continuously changes the pattern of questions asked in the examination, it is recommended that the student should work on his/her conceptual clarity and analytical ability, with respect to both static and current affairs part.

2. In depth study of the topics of current affairs is required, which is not only useful from prelims point of view but also from mains point of view.

3. However, some questions can be solved using common sense and applying knowledge in the examination. The skill to solve objective questions should be developed through regular practice of MCQs.

4. UPSC frequently varies the weightage of the major areas of syllabus, thus it is advised to give equal focus to all areas.

5. Student should also work on his/her temperament and exam management skills. Since UPSC is giving options too close and playing with words (e.g. Arthur Wellesley instead of Richard Wellesley), it is important to read each and every word/statement carefully.

6. CUT-OFF for this year will be lower than last year most likely to be in the range of 97-102 marks. But seeing the unpredictable nature of examination, it is advised that candidates who are getting marks in 90s should continue preparations for the mains examination.

Note:1. Applied Current means inspired by current events or mix of current and traditional.2. Applied traditional means conceptual and application based questions from traditional part.

Page 3: CSE-PRELIMS 2018 • PAPER-1 · 2 | Detailed of Solutions of CSE-Prelims:2018 • Paper-1 Analysis of 2018 Prelims: 1. The question paper was difficult as compared to the last year’s

Detailed of Solutions of CSE-Prelims:2018 • Paper-1 | 3

Page 4: CSE-PRELIMS 2018 • PAPER-1 · 2 | Detailed of Solutions of CSE-Prelims:2018 • Paper-1 Analysis of 2018 Prelims: 1. The question paper was difficult as compared to the last year’s

4 | Detailed of Solutions of CSE-Prelims:2018 • Paper-1

Next IAS Prelims Test Series Current affairs Magazine

1

Consider the following events :

1. The first democratically elected communist partygovernment formed in a State in India.2. India’s then largest bank, ‘Imperial Bank of India’, wasrenamed ‘State Bank of India’.3. Air India was nationalised and became the nationalcarrier.4. Goa became a part of independent India.

Which of the following is the correct chronologicalsequence of the above events?(a) 4-1-2-3(b) 3-2-1-4(c) 4-2-1-3(d) 3-1-2-4

b Post Indepenence

Tata Sons set up Tata Airlines in 1932. In1946, Tata Airlines became a publiccompany and was renamed Air India. In1953, air India was nationalized andbecame the national carrier.SBI is the oldest and largest public sectorbank in the Indian banking sector. TheBank of Calcutta, the Bank of Bombayand the Bank of Madras were merged byan act of legislature to form the ImperialBank of India in 1921. In 1955, theImperial Bank of India was nationalizedand renamed as SBI.In 1957, the Communist Party of India(CPI), led by EMS Namboodiripadbecame one of the first democraticallyelected Communist governments in theworld, winning elections Kerala.The Portuguese invaded Goa in 1510,and ruled it for over four centuries. Goawas liberated from Portuguese rule in1961.

https://rbidocs.rbi.org.in/rdocs/content/PDFs/90028.pdfLaxmikanth 5th edition page 5.5http://ncert.nic.in/ncerts/l/leps202.pdf,http://www.airindia.in/timeline.htmhttps://www.nseindia.com/content/corporate/eq_SBIN_base.pdf

Difficult

Magazine (Current affairs Made Easy) Statement 4 is mentioned in May 2018 edition (Page 38): “Although Goa got its freedom on 19 December 1961 from Portuguese, the first blow for Goa’s liberation was struck on 18 June 1946 by Ram Manohar Lohia”.

2

Right to Privacy is protected as an intrinsic part of Rightto Life and Personal Liberty. Which of the following inthe Constitution of India correctly and appropriately implythe above statement?

(a) Article 14 and the provisions under the 42ndAmendment to the Constitution(b) Article 17 and the Directive Principles of State Policyin Part IV(c) Article 21 and the freedoms guaranteed in Part III(d) Article 24 and the provisions under the 44thAmendment to the Constitution

c Fundamental Rights

In 2017, A nine-judge bench of theSupreme Court ruled that Indians enjoy afundamental right to privacy, that it isintrinsic to life and liberty and thus comesunder Article 21 of the Indian constitution.

Page 175 5th edition Laxmikanthhttps://www.thehindubusinessline.com/news/privacy-is-intrinsic-to-life-and-liberty-rules-apex-court/article9829145.ece

Easy

Test 18, Explanation 94:The Supreme Court has reaffirmed its judgementin the Menaka case in the subsequent cases. Ithas declared the following rights as part of Article21: (a) Right to live with human dignity. (b) Rightto decent environment including pollution freewater and air and protection against hazardousindustries. (c) Right to livelihood. (d) Right toprivacy. (e) Right to shelter. (f) Right to health.(g) Right to free education up to 14 years of age.(h) Right to free legal aid. (i) Right againstsolitary confinement. (j) Right to speedy trial. (k)Right against handcuffing. (l) Right againstinhuman treatment. (m) Right against delayedexecution. (n) Right to travel abroad. (o) Rightagainst bonded labour. (p) Right againstcustodial harassment. (q) Right to emergencymedical aid. (r) Right to timely medical treatmentin government hospital. (s) Right not to be drivenout of a state. (t) Right to fair trial. (u) Right ofprisoner to have necessities of life. (v) Right ofwomen to be treated with decency and dignity.(w) Right against public hanging. (x) Right tohearing. (y) Right to information. (z) Right toreputation.

Next IAS ContentQ. No QUESTION Answer Topic/Area Explanation Source Level

Page 5: CSE-PRELIMS 2018 • PAPER-1 · 2 | Detailed of Solutions of CSE-Prelims:2018 • Paper-1 Analysis of 2018 Prelims: 1. The question paper was difficult as compared to the last year’s

Detailed of Solutions of CSE-Prelims:2018 • Paper-1 | 5

3

Consider the following :

1. Areca nut 2. Barley 3. Coffee 4. Finger millet 5. Groundnut 6. Sesamum 7. Turmeric

The Cabinet Committee on Economic Affairs hasannounced the Minimum Support Price for which of theabove?(a) 1, 2, 3 and 7 only(b) 2, 4, 5 and 6 only(c) 1, 3, 4, 5 and 6 only(d) 1, 2, 3, 4, 5, 6 and 7

b Schemes and Policies

The commodities included in the listdeclared by Cabinet Committee onEconomic Affairs are-

Kharif Crops: Paddy Common, Paddy(F)/Grade'A', Jowar-Hybrid, Jowar-Maldandi, Bajra, Ragi, Maize, Tur,Moong, Urad, groundnut, Sunflower seed,Soyabean black, soyabean yellow,Sesamum, Nigerseed, Medium staplecotton, long staple cotton,

Rabi Crops: Wheat, barley, Gram, Lentil,Mustard, Safflower

Other Commercial Crops: Jute,Sugarcane, Copra (Milling), Copra (Ball)

https://cacp.dacnet.nic.in/ViewContents.aspx?Input=1&PageId=36&KeyId=0http://vikaspedia.in/agriculture/market-information/minimum-support-price

Medium

4

In which one of the following States is Pakhui WildlifeSanctuary located?

(a) Arunachal Pradesh(b) Manipur(c) Meghalaya(d) Nagaland

a Protected Areas

Pakhui Wildlife Sanctuary (Pakke) lies inthe foothills of the Eastern Himalaya inthe East Kameng District of ArunachalPradesh. It was declared a sanctuary in1977, and was earlier part of the KhellongForest Division. It was declared a tigerreserve in 2002 based on a proposal in1999.

http://www.arunachaltourism.com/pakhui-wildlife-sanctuary.phphttp://wiienvis.nic.in/Database/biodiversity_pas_8557.aspx

Easy

Test 7, Explanation 24:Located on the border with Assam and coveringan area of 862 sq km, Pakke was part of theKhellong Forest Division and was declared agame sanctuary in 1977. The sanctuary was laterdeclared a Tiger Reserve in 2002. Statement 1 iscorrect: It is the home of four hornbill species aswell as the endangered whitewinged wood duck.Other notable bird species include the collaredand long-tailed broadbills, white-browedshortwing, yellow-bellied flycatcher-warbler, crow-billed drongo, emerald cuccko, ibis bill, greypeacock-pheasant. Statement 2 is correct:Pakke comes under the Eastern HimalayaBiodiversity Hotspot, and is home to over 2,000species of plants, 300 species of birds, and over30 species each of mammals, reptiles andamphibians. Test 16, Explanation 15:Pakke Tiger reserve is located in ArunachalPradesh.Test 17, Explanation 36

Page 6: CSE-PRELIMS 2018 • PAPER-1 · 2 | Detailed of Solutions of CSE-Prelims:2018 • Paper-1 Analysis of 2018 Prelims: 1. The question paper was difficult as compared to the last year’s

6 | Detailed of Solutions of CSE-Prelims:2018 • Paper-1

5

With reference to India’s satellite launch vehicles,consider the following statements :1. PSLVs launch the satellites useful for Earthresources monitoring whereas GSLVs are designedmainly to launch communication satellites.2. Satellites launched by PSLV appear to remainpermanently fixed in the same position in the sky, asviewed from a particular location on Earth.3. GSLV Mk III is a four-stage launch vehicle with thefirst and third stages using solid rocket motors, and thesecond and fourth stages using liquid rocket engines.

Which of the statements given above is/are correct?(a) 1 only(b) 2 and 3(c) 1 and 2(d) 3 only

a Technology

Both PSLV (Polar Satellite LaunchVehicle) and GSLV (GeosynchronousSatellite Launch Vehicle) are the satellite-launch vehicles (rockets) developed byISRO. Statement 1 is correct: PSLV is designedmainly to deliver the “earth-observation”or “remote-sensing” satellites. The GSLVis designed mainly to deliver thecommunication-satellites to the highlyelliptical (typically 250 x 36000 Km)Geosynchronous Transfer Orbit (GTO).

Statement 2 is not correct: Due to theirgeo-synchronous nature, the satelliteslaunched by GSLV in these orbits appearto remain permanently fixed in the sameposition in the sky, as viewed from aparticular location on Earth, thus avoidingthe need of a tracking ground antennaand hence are useful for thecommunication applications.Statement 3 is not correct: GSLV Mk IIIis a three-stage heavy lift launch vehicledeveloped by ISRO. The vehicle has twosolid strap-ons, a core liquid booster anda cryogenic upper stage.

https://www.isro.gov.in/launchers, https://www.isro.gov.in/launchers/gslv-mk-iiihttp://www.thehindu.com/sci-tech/science/what-is-the-difference-between-gslv-and-pslv/article6742299.ece

Medium

Test 1, Question 93:With reference to the recently launched GSLVMARK III by ISRO, Consider the followingstatements: 1. GSKV-Mk III is capable of launchingfourtonne satellites in the GeosynchronousTransfer Orbit (GTO). 2. GSLV-Mk III is a two-stage vehicle with acryogenic stage (C-25). 3. It was earlier named as Launch Vehicle Mark-3 or LVM-3. Which of the statements given above arecorrect? (a) 1 and 2 only (b) 1 and 3 only (c) 2 and 3 only (d) 1, 2 and 3Explanation:Statement 1 is correct:GSLV MKIII is capable oflaunching 4 tonne satellites in theGeosynchronous transfer orbit (GTO). Statement2 is not correct: GSLV MKIII is a 3-stage vehiclewith a cryogenic stage. GSLV MK III is a threestage heavy lift launch vehicle developed byISRO. The vehicle has two solid strap-ons, acore liquid booster and a cryogenic upper stageStatement 3 is correct: It was earlier named asLaunch Vehicle MARK-3 or LVM-3.

Magazine (Current affairs Made Easy) Statement 3 is mentioned in July-August-September, 2017 edition (Page 142): “GSLV MK III is a three-stage heavy lift launch vehicle developed by ISRO. The vehicle has two solid strap-ons, a core liquid booster and a cryogenic upper stage.

6

With reference to the governance of public sectorbanking in India, consider the following statements:

1. Capital infusion into public sector banks by theGovernment of India has steadily increased in the lastdecade.2. To put the public sector banks in order, the merger ofassociate banks with the parent State Bank of India hasbeen affected.

Which of the statements given above is/are correct?(a) 1 only(b) 2 only(c) Both 1 and 2(d) Neither 1 nor 2

b Banking Sector

Government infuses capital into PSUbanks to support credit expansion and tohelp them tide over losses resulting fromprovisions that are to be made for non-performing assets (NPAs).Statement 1 is not correct: Capital infusion into public sector banks by thegovernment of India has not steadilyincreased in the last decade. There areinstances where the there was a dip in the value of amount given by the governmentfor capital infusion of public sector bankseg- when we see the trend we can seethat for the year 2009-10 the amount was1200 crore, for 2010-11 it was 20117.23,and for the year 2011-12 the amount was12000 crore, from here we can see thatthe total amount increased from 2009-10to 2010-11, then from 2010-11 to 2011-12, the amount again decreased. Statement 2 is correct: To put thepublic sector banks in order, themerger of associate banks with theparent State Bank of India has beenaffected.

https://rbi.org.in/SCRIPTS/PublicationsView.aspx?id=18060,https://www.rbi.org.in/scripts/BS_PressReleaseDisplay.aspx?prid=39884,http://pib.nic.in/newsite/PrintRelease.aspx?relid=158491https://data.gov.in/catalog/capital-infusion-public-sector-banks-government

Difficult

Page 7: CSE-PRELIMS 2018 • PAPER-1 · 2 | Detailed of Solutions of CSE-Prelims:2018 • Paper-1 Analysis of 2018 Prelims: 1. The question paper was difficult as compared to the last year’s

Detailed of Solutions of CSE-Prelims:2018 • Paper-1 | 7

7

Consider the following items:

1. Cereal grains hulled2. Chicken eggs cooked3. Fish processed and canned4. Newspapers containing advertising materialWhich of the above items is/are exempted under GST(Goods and Services Tax) ?(a) 1 only(b) 2 and 3 only(c) 1, 2 and 4 only(d) 1, 2, 3 and 4

c Tax Structure

Out of the given items, Cereal grainshulled, Chicken eggs cooked andNewspapers containing advertisingmaterial are exempted under the GST taxregime.

http://gstcouncil.gov.in/sites/default/files/NOTIFICATION%20PDF/goods-rates-booklet-03July2017.pdf

Difficult

8

Consider the following statements :

1. The definition of “Critical Wildlife Habitat” isincorporated in the Forest Rights Act, 2006.2. For the first time in India, Baigas have been givenHabitat Rights.3. Union Ministry of Environment, Forest and ClimateChange oflicially decides and declares Habitat Rights forPrimitive and Vulnerable Tribal Groups in any part ofIndia.

Which of the statements given above is/are correct?(a) 1 and 2 only(b) 2 and 3 only(c) 3 only(d) 1, 2 and 3

a Governance

Statement 1 is correct: The term"Critical Wildlife Habitat" is incorporatedin the Forests Rights Act, 2006.Statement 2 is correct: Living in thedense sal forests of Maikal Hills, wherestreams intersect their homesteads, theBaigas in Dindori district of MadhyaPradesh have gained the right to theirhabitat. This is for the first time habitatrights have been given under the ForestRights Act of 2006. For the Baigas, it is aPyrrhic victory that has come after astruggle for more than a century.Statement 3 is not correct: Ministry ofTribal Affairs officially decides anddeclares Habitat Rights for Primitive andVulnerable tribal Groups in any part onIndia.

http://pib.nic.in/newsite/PrintRelease.aspx?relid=69806,https://tribal.nic.in/pvtg.aspx

Medium

Test 17, Question no. 23:With reference to the ‘Forest Rights Act, 2006’ inIndia, consider the following statements: 1. The Act confers ownership rights over minorforest produce to forest dwelling tribes. 2. The Ministry of Environment, Forest andClimate Change (MoEFCC) is the nodalagency for the implementation of the Act of2006. 3. It extends to the whole of India except theState of Jammu and Kashmir. Which of the statements given above is/arecorrect? (a) 1 and 2 only (b) 2 and 3 only H12(c) 1 and 3 only (d) 1, 2 and 3 Solution:This Act is also called the Scheduled Tribes andOther Traditional Forest Dwellers (Recognition ofForest Rights) Act, 2006. Statement 1 iscorrect: This Act not only recognizes the rightsto hold and live in the forest land under theindividual or common occupation for habitation orfor self-cultivation for livelihood, but also grantsseveral other rights to ensure their control overforest resources which, inter-alia, include right ofownership, access to collect, use and dispose ofminor forest produce, community rights such asnistar; habitat rights for primitive tribal groupsand pre-agricultural communities; right to protect,regenerate or conserve or manage anycommunity forest resource which they have beentraditionally protecting and conserving forsustainable use. Statement 2 is not correct: The Ministry ofTribal Affairs is the nodal agency for theimplementation of the Act of 2006. Statement 3is correct: It extends to the whole of India exceptthe State of Jammu and Kashmir. Statement 3 is correct: It extends to the wholeof India except the State of Jammu and Kashmir

Page 8: CSE-PRELIMS 2018 • PAPER-1 · 2 | Detailed of Solutions of CSE-Prelims:2018 • Paper-1 Analysis of 2018 Prelims: 1. The question paper was difficult as compared to the last year’s

8 | Detailed of Solutions of CSE-Prelims:2018 • Paper-1

9

Consider the following :

1. Birds 2. Dust blowing 3. Rain 4. Wind blowing

Which of the above spread plant diseases?(a) 1 and 3 only(b) 3 and 4 only(c) 1, 2 and 4 only(d) 1, 2, 3 and 4

d Ecology

Most disease-causing viruses are carriedand transmitted naturally by insects andmites, which are called vectors of thevirus. Slugs, snails, birds, rabbits, anddogs also transmit a few viruses, but thisis not common. Diseases that attack theleaves of a plant are primarily spread bywind, but they can also move to nearbyplants by taking a ride on splashing waterdroplets from rain or irrigation. Dustblowing also plays role in spreading plantdiseases.

https://www.britannica.com/science/plant-disease/Transmissionhttps://www.sciencefriday.com/educational-resources/how-do-diseases-spread-between-plants/

Easy

10

With reference to organic farming in India, consider thefollowing statements:

1.‘The National ‘Programme for Organic Production’(NPOP) is operated under the guidelines and‘ directionsof the Union Ministry of Rural Development.2.‘The Agricultural and Processed Food Product ExportDevelopment Authority‘ (APEDA) functions as theSecretariat for the implementation of NPOP.3. Sikkim has become India’s first fully organic State.

Which of the statements given above is/are correct?

(a) 1 and 2 only(b) 2 and 3 only(c) 3 only(d) 1, 2 and 3

b Schemes and Policies

Statement 1 is not correct: Ministry ofCommerce has implemented the NationalProgramme for Organic Production(NPOP) since 2001. Statement 2 is correct: APEDA shallfunction as the Secretariat for theimplementation of the NPOP. Statement 3 is correct: Sikkim becameIndia's first fully organic state in July 2016.

http://apeda.gov.in/apedawebsite/organic/index.htm, http://pib.nic.in/PressReleseDetail.aspx?PRID=1508118,http://pib.nic.in/newsite/PrintRelease.aspx?relid=155000http://apeda.gov.in/apedawebsite/organic/organic_contents/Chapter_2.pdfhttp://www.thehindu.com/news/national/Sikkim-becomes-India%E2%80%99s-first-organic-state/article13999445.ece

Medium

Magazine (Current affairs Made Easy)Statement 3 is mentioned in February 2018 edition (Page 76)

Page 9: CSE-PRELIMS 2018 • PAPER-1 · 2 | Detailed of Solutions of CSE-Prelims:2018 • Paper-1 Analysis of 2018 Prelims: 1. The question paper was difficult as compared to the last year’s

Detailed of Solutions of CSE-Prelims:2018 • Paper-1 | 9

11

Regarding Money Bill, which of the following statementsis not correct?

(a) A bill shall be deemed to be a Money Bill if it containsonly provisions relating to imposition, abolition,remission, alteration or regulation of any tax.(b) A Money Bill has provisions for the custody of theConsolidated Fund of India or the Contingency Fund ofIndia.(c) A Money Bill is concerned with the appropriation ofmoney out of the Contingency Fund of India.(d) A Money Bill deals with the regulation of borrowing ofmoney or giving of any guarantee by the Government ofIndia.

c Parliament

Article 110 of the Constitution deals withthe definition of money bills. It states thata bill is deemed to be a money bill if itcontains ‘only’ provisions dealing with allor any of the following matters: 1. Theimposition, abolition, remission, alterationor regulation of any tax (Option a iscorrect); 2. The regulation of theborrowing of money by the Uniongovernment (Option d is correct); 3. Thecustody of the Consolidated Fund of Indiaor the contingency fund of India, thepayment of moneys into or the withdrawalof money from any such fund (Option bis correct); 4. The appropriation ofmoney out of the Consolidated Fund ofIndia; 5. Declaration of any expenditurecharged on the Consolidated Fund ofIndia or increasing the amount of anysuch expenditure; 6. The receipt of money on account of the Consolidated Fund ofIndia or the public account of India or thecustody or issue of such money, or theaudit of the accounts of the Union or of astate; or 7. Any matter incidental to any ofthe matters specified above

Laxmikanth Fifth edition, page no. 22.19 Easy

Test 8, Explanation 27:Article 110 of the Constitution deals with thedefinition of money bills. It States that a bill isdeemed to be a money bill if it contains ‘only’provisions dealing with all or any of the followingmatters: 1. The imposition, abolition, remission, alterationor regulation of any tax; 2. The regulation of the borrowing of money bythe Union government; 3. The custody of the Consolidated Fund of Indiaor the contingency fund of India, the payment ofmoneys into or the withdrawal of money from anysuch fund; 4. The appropriation of money out of theConsolidated Fund of India; 5. Declaration of any expenditure charged on theConsolidated Fund of India or increasing theamount of any such expenditure; 6. The receipt of money on account of theConsolidated Fund of India or the public accountof India or the custody or issue of such money, orthe audit of the accounts of the Union or of astate; or 7. Any matter incidental to any of the mattersspecified above. If any question arises whether abill is a money bill or not, the decision of theSpeaker of the Lok Sabha is final.

12

With reference to the election of the President of India,consider the following statements:1. The value of the vote of each MLA varies from Stateto State.2. The value of the vote of MPs of the Loksabha is morethan the value of the vote of MPs of the Rajyasabha.Which of the following statements given above is/arecorrect?

(a) 1 only(b) 2 only(c) Both 1 and 2(d)Neither 1 nor 2

a Parliament and State Legislature

Statement 1 is correct: The formula fordetermining the number of votes held byan MLA is:Statement 2 is not correct: The value ofan MP's vote is calculated by dividing thetotal value of all MLAs' votes by thenumber of MPs. The formula fordetermining the number of votes held byan MP is:

Laxmikanth Fifth edition, page no. 17.3 Easy

Magazine (Current affairs Made Easy) July-August-September, 2017 edition (Page 83)

13

In the Indian context, what is the implication of ratifying the‘Additional Protocol’ with the ‘International Atomic EnergyAgency (IAEA)’?(a) The civilian nuclear reactors come under IAEAsafeguards.(b)The military nuclear installations come under theinspection of IAEA(c)The country will have the privilege to buy uranium fromthe Nuclear Suppliers Group (NSG).(d) The country automatically becomes a member of theNSG.

a Institutions and Agencies

An Additional Protocol (AP) to theSafeguards Agreement between theGovernment of India and the IAEA for theApplication of Safeguards to CivilianNuclear Facilities entered into force on 25July 2014.

https://www.iaea.org/newscenter/news/indias-additional-protocol-enters-force

Medium

Total population of the state or union territoryValue of an MLA vote =

Total number of elected numbers of the Legislative Assembly ×100

Total sum of vote value of elected members of all the Legislative AssembliesValue of an MLA vote =

The sum elected members of both the houses of Parliament

Page 10: CSE-PRELIMS 2018 • PAPER-1 · 2 | Detailed of Solutions of CSE-Prelims:2018 • Paper-1 Analysis of 2018 Prelims: 1. The question paper was difficult as compared to the last year’s

10 | Detailed of Solutions of CSE-Prelims:2018 • Paper-1

14

Consider the following countries :

1. Australia2. Canada3. China4. India5. Japan6. USA

Which of the above are among the ‘free-trade partners’of ASEAN ?(a) 1, 2, 4 and 5(b) 3, 4, 5 and 6(c) 1, 3, 4 and 5(d) 2, 3, 4 and 6

cInternational

Organisations/Partnerships

The Association of Southeast AsianNations (ASEAN) has free tradeagreements with six partners namelyPeople’s Republic of China (ACFTA),Republic of Korea (AKFTA), Japan(AJCEP), India (AIFTA) as well asAustralia and New Zealand (AANZFTA).

http://asean.org/?static_post=rcep-regional-comprehensive-economic-partnership

Easy

Test 22, Explanation 66:The RCEP is a proposed mega-regional FTAinvolving the 10 member countries of ASEANand its six FTA partners including India, China,Japan, South Korea, Australia and New Zealand.In August 2012, the 16 Economic Ministersendorsed the Guiding Principles and Objectivesfor Negotiating the Regional ComprehensiveEconomic Partnership. The RCEP negotiationswere launched by Leaders from 10 ASEANMember States (Brunei Darussalam, Cambodia,Indonesia, Lao PDR, Malaysia, Myanmar, thePhilippines, Singapore, Thailand and Viet Nam)and six ASEAN FTA partners (Australia,People’s Republic of China, India, Japan,Republic of Korea, and New Zealand) during the21st ASEAN Summit and Related Summits inPhnom Penh, Cambodia in November 2012.

Magazine (Current affairs Made Easy)March 2018 edition (Page 9)

15

With reference to the ‘Global Alliance for Climate-SmartAgriculture (GACSA)’, which of the following statementsis/are correct ?

1. GACSA is an outcome of the Climate Summit held inParis in 2015.2. Membership of GACSA does not create any bindingobligations.3. India was instrumental in the creation of GACSA.

Select the correct answer using the code given below :(a) 1 and 3 only(b) 2 only(c) 2 and 3 only(d) 1, 2 and 3

b Institutions and Agencies

Statement 1 is not correct: The GlobalAlliance for Climate Smart Agriculture(GACSA) was initiated and established in2014, in UN Climate Summit 2014.

Statement 2 is correct: Membership ofGACSA does not create any bindingobligations.

Statement 3 is not correct: India as acountry is not a member of GACSA.

http://www.fao.org/gacsa/about/en/https://www.norad.no/en/front/thematic-areas/climate-change-and-environment/food-security/right-to-food/https://www.iatp.org/blog/201509/what%E2%80%99s-wrong-with-%E2%80%9Cclimate-smart%E2%80%9D-agriculture

Medium

16

Which of the following is/are the aim/aims of “DigitalIndia” Plan of the Government of India ?

1. Formation of India’s own Internet companies likeChina did.2. Establish a policy framework to encourage overseasmultinational corporations that collect Big Data to buildtheir large data centres within our national geographicalboundaries.3. Connect many of our villages to the Internet and bringWi-Fi to many of our schools, public places and majortourist centres.

Select the correct answer using the code given below :(a) 1 and 2 only(b) 3 only(c) 2 and 3 only(d) 1, 2 and 3

b Schemes and Policies

Digital India aims to provide the muchneeded thrust to the nine pillars of growthareas, namely Broadband Highways,Universal Access to Mobile Connectivity,Public Internet Access Programme, e-Governance: Reforming Governmentthrough Technology, e-Kranti - ElectronicDelivery of Services, Information for All,Electronics Manufacturing, IT for Jobsand Early Harvest Programmes. Each ofthese areas is a complex programme initself and cuts across multiple Ministriesand Departments.

http://vikaspedia.in/e-governance/digital-india/nine-pillars-of-digital-indiahttp://www.digitalindia.gov.in/content/programme-pillars

Easy

Page 11: CSE-PRELIMS 2018 • PAPER-1 · 2 | Detailed of Solutions of CSE-Prelims:2018 • Paper-1 Analysis of 2018 Prelims: 1. The question paper was difficult as compared to the last year’s

Detailed of Solutions of CSE-Prelims:2018 • Paper-1 | 11

17

Consider the following pairs :

Towns sometimes mentioned in news Country1. Aleppo Syria2. Kirkuk Yemen3. Mosul Palestine4. Mazar-i-sharif Afghanistan

Which of the pairs given above are correctly matched?(a) 1 and 2(b) 1 and 4(c) 2 and 3(d) 3 and 4

b MappingAleppo is located in Syria, Kirkuk andMosul are in Iraq and Mazar-i-sharif is inAfghanistan.

http://www.bbc.com/news/world-middle-east-38132163http://www.thehindu.com/todays-paper/tp-international/mazar-i-sharif-afghanistan-april-7-2018-afp/article23469762.ecehttp://www.thehindu.com/news/international/iraqi-forces-complete-takeover-of-kirkuk-province-after-clashing-with-kurds/article19888096.ecehttps://www.aljazeera.com/indepth/inpictures/mosul-body-collectors-180530113119652.html

Easy

Test 4, Explanation99Mosul is a major city in northern Iraq. It is locatedsome 400 km north of Baghdad. Mosul stands onthe west bank of the Tigris, opposite the ancientAssyrian city of Nineveh on the east bank. Iraqiforces launched a military campaign to oust thehardline extremists from Mosul and surroundingNineveh province. Idlib is a city in northwestern Syria, capital of theIdlib Governorate, 59 kilometers southwest ofAleppo. Recently, A Turkish militaryreconnaissance team has been deployed to thenorthern Syrian province of Idlib ahead of theestablishment of a “de-escalation zone” there.

Magazine (Current affairs Made Easy)Mosul is mentioned in January, 2018 edition (Page 79): “At the peak of its influence, the IS controlled almost a third of Iraq, including Mosul, its second largest city”.

18

In the Federation established by The Government ofIndia Act of 1935, residuary powers were given to the

(a)Federal Legislature(b) Governor General(c)Provincial Legislature(d) Provincial Governors

b Making of Constitution

In the federation established by theGovernment of India Act 1935, residuarypowers were given to the GovernorGeneral to decide in his sole discretion asto under which list a particular subject fell.

Laxmikanth Fifth edition, page no. 1.7BL Grover Page 404

Easy

19

Consider the following statements :

1. The Speaker of the Legislative Assembly shall vacatehis/her office if he/she ceases to be a member of theAssembly.2. Whenever the Legislative Assembly is dissolved, theSpeaker shall vacate his/her office immediately.

Which of the statements given above is/are correct ?(a) 1 only(b) 2 only(c) Both 1 and 2(d) Neither 1 nor 2

a Parliament

Statement 1 is correct: The Speakerremains in office during the life of theassembly. However, he vacates his officeearlier in any of the following three cases:1. if he ceases to be a member of theassembly; 2. if he resigns by writing to thedeputy speaker; and 3. if he is removedby a resolution passed by a majority of allthe then members of the assembly. Sucha resolution can be moved only aftergiving 14 days advance notice.Statement 2 is not correct: Wheneverthe Lok Sabha is dissolved, the Speakerdoes not vacate his office and continuestill the newly elected Lok Sabha meets.

Laxmikanth Fifth edition, page no 22.7 Easy

Test 19, Explanation 98:Speaker is removed by a resolution passed by amajority of all the members of the Lok Sabha.Such a resolution can be moved only after giving14 days’ advance notice. When a resolution forthe removal of the Speaker is underconsideration of the House, he cannot preside atthe sitting of the House, though he may bepresent. However,he can speak and take part in the proceedingsof the House at such a time and vote in the firstinstance, though not in the case of an equalityof votes. It should be noted here that,wheneverthe Lok Sabha is dissolved, the Speaker doesnotvacate his office and continues till thenewlyelectedLok Sabha meets.

20

Which one of the following reflects the most appropriaterelationship between law and liberty ?

(a) If there are more laws, there is less liberty.(b) If there are no laws, there is no liberty.(c) If there is liberty laws have to be made by the people.(d) If laws are changed too often, liberty is in danger.

a Political Theory

Freedom is said to exist when externalconstraints on the individual are absent.In terms of this definition an individualcould be considered free if he/she is notsubject to external controls or coercionand is able to make independentdecisions and act in an autonomous way.

http://www.ncert.nic.in/ncerts/l/keps102.pdf Difficult

Page 12: CSE-PRELIMS 2018 • PAPER-1 · 2 | Detailed of Solutions of CSE-Prelims:2018 • Paper-1 Analysis of 2018 Prelims: 1. The question paper was difficult as compared to the last year’s

12 | Detailed of Solutions of CSE-Prelims:2018 • Paper-1

21

Consider the following statements :

1. No criminal proceedings shall be instituted against theGovernor of a State in any court during his term of office.2. The emoluments and allowances of the Governor of aState shall not be diminished during his term of office.

Which of the statements given above is/are correct?(a) 1 only(b) 2 only(c) Both 1 and 2(d) Neither 1 nor 2

c Governor

Statement 1 is correct: No criminalproceedings whatsoever shall beinstituted or continued against thePresident, or theGovernor of a State, in any court duringhis term ofoffice.Statement 2 is correct: Theemoluments and allowances of theGovernorshall not be diminished during his term ofoffice.

Bare Act Article 361: http://www.legislative.gov.in/sites/default/files/coi-4March2016.pdfBare Act Article 158

Easy

Test 8, Question 40: Which of the following ‘Privileges’ is/areendowed with Governor during the term of hisoffice? 1. He enjoys personal immunity from legalliability for his official acts. 2. He is immune from any criminalproceedings even in respect for his personalacts. 3. He holds his office for a fixed term of 5 years. Select the correct answer using the code givenbelow: (a) 1 only (b) 1 and 2 only (c) 2 and 3 only (d) 1, 2 and 3 Solution:Like the President, the Governor is also entitledto a number of privileges and immunities.Statement 1 and 2 are correct: He enjoyspersonal immunity from legal liability for hisofficial acts. During his term of office, he isimmune from any criminal proceedings, even inrespect of his personal acts. He cannot bearrested or imprisoned. However, after giving twomonths’ notice, civil proceedings can beinstituted against him during his term of office inrespect of his personal acts.Statement 3 is not correct: A Governor holdsoffice for a term of five years from the date onwhich he enters upon his office. However, thisterm of five years is subject to the pleasure of thePresident. Further, he can resign at any time byaddressing a resignation letter to the President.

Page 13: CSE-PRELIMS 2018 • PAPER-1 · 2 | Detailed of Solutions of CSE-Prelims:2018 • Paper-1 Analysis of 2018 Prelims: 1. The question paper was difficult as compared to the last year’s

Detailed of Solutions of CSE-Prelims:2018 • Paper-1 | 13

22

The well-known painting “Bani Thani” belongs to the

(a) Bundi school(b) Jaipur school(c) Kangra school(d) Kishangarh school.

d Paintings

Bani Thani Paintings were in newsrecently because there will be a display offamous Bani-Thani paintings ofKishangarh style in the galleries and hallsof the newly built Kishangarh airport. The painting of Nihalchand of Kishangarh of Rajasthani School has a special placein the treasure of Indian miniaturepaintings. It is often compared toMonalisa supposed to epitomise the idealof womanhood.

https://timesofindia.indiatimes.com/city/jaipur/bani-thani-paintings-to-welcome-passengers-at-kishangarh-airport/articleshow/58451262.cms

Medium

Test 10, Question 89:This painting by Nihalchand has a special placein the treasure of Indian Miniature painting. Thestyle is characterized by emphatic lengthening ofthe eyes and nose. Faces are mostly in profilewith a very stylized eyebrow and smile on thelips. The above description refers to which one ofthe following paintings? (a) Bani Thani painting of Rajasthan. (b) Mughal painting from Ain-i-akbari. (c) Nayak-nayika painting of Kangra. (d) Devi series of Basohli in Jammu andKashmir. Solution:The painting of Nihalchand of Kishangarh of Rajasthani chool has a special place in thetreasure of Indian miniature paintings. The styleis characterized by emphatic lengthening of theeyes and nose. Faces are mostly in profile with avery stylized eyebrow and smile on the lips. Thepainter was inspired by the poem of Raja SawantSingh., who describes the beauty of a lady called‘Bani-Thani’ in his poem. The beauty of thiswoman is symbolically used to describe thebeauty of Sri Radha. Her smile is supposedlyenigmatic; her eyes reflect the ideal femininebeauty of classic Sanskrit Literature. She ispainted as an ideal of Indian womanhood andcan easily be compared to Monalisa supposed toeptomise the ideal of womanhood.

23

What is “Terminal High Altitude Area Defense(THAAD)”, sometimes seen in the news ?

(a) An Israeli radar system(b) India’s indigenous anti-missile programme(c)An American anti-missile system(d) A defence collaboration between Japan and SouthKorea

c Defence Technology

THAAD is an addition to the UnitedStates’ anti-ballistic missile/interceptortoolkit. It entered production in 2008 andis primarily tasked with taking outthreatening ballistic missiles in what’sknown as their “terminal” phase (the ‘T’ inthe acronym).

https://thediplomat.com/2016/02/what-is-thaad-what-does-it-do-and-why-is-china-mad-about-it/

Easy

Magazine (Current affairs Made Easy)October 2017 edition (Page 24)

Page 14: CSE-PRELIMS 2018 • PAPER-1 · 2 | Detailed of Solutions of CSE-Prelims:2018 • Paper-1 Analysis of 2018 Prelims: 1. The question paper was difficult as compared to the last year’s

14 | Detailed of Solutions of CSE-Prelims:2018 • Paper-1

24

With reference to cultural history of India, consider thefollowing statements :

1. Most of the Tyagaraja Kritis are devotional songs inpraise of Lord Krishna.2. Tyagaraja created several new ragas.3. Annamacharya and Tyagaraja are contemporaries.4. Annamacharya kirtanas are devotional songs inpraise of Lord Venkateshwara.

Which of the statements given above are correct?(a)1 and 3 only(b) 2 and 4 only(c)1, 2 and 3(d) 2, 3 and 4

b Personalities/Culture

Statement 1 is not correct: Most ofTyagaraja’s songs were in praise of LordRama.

Statement 2 is correct: Tyagarajacreated several new ragas.

statement 3 is not correct: :SriTalapakka Annamacharya of Talapakkavillage in Cuddapah district was asaintcomposer of the15th century, whileTyagaraja was born in 1767, and recentlyhis 250th birth anniversary wascelebrated.Statement 4 is correct: Annamacharyakirtanas are devotional songs in praise ofLord Venkateshwara.

https://www.britannica.com/biography/Tyagaraja

https://books.google.co.in/books?isbn=818495445X

Difficult

25

Which of the following are regarded as the main featuresof the “Rule of Law” ?

1. Limitation of powers2. Equality before law3. People’s responsibility to the Government4. Liberty and civil rights

Select the correct answer using the code given below :(a) 1 and 3 only(b) 2 and 4 only(c) 1, 2 and 4 only(d) 1, 2, 3 and 4

c Governance

Rule of law includes limitation of power,equality before law and liberty and civilrights. What the rule of law means is thatall laws apply equally to all citizens of thecountry and no one can be above the law.Neither a government official, nor awealthy person nor even the President ofthe country is above the law.

http://ncert.nic.in/ncerts/l/iess406.pdfhttp://ncert.nic.in/ncerts/l/hess304.pdfhttp://www.ncert.nic.in/ncerts/l/keps102.pdf

Medium

Magazine (Current affairs Made Easy)following below are the excerpts from April 2018 edition (Page 41)• Principles of the Rule of Law: The WJP uses a orking definition of the rule of law based on four universa principles, derived from internationally accepted standards: (1) Accountability, (2) Just Laws, (3) Open Government and (4) Accessible & Impartial Dispute Resolution.• Methodology: The Index measures countries’ rule of law performance across eight factors: (1) Constraints on Government Powers, (2) Absence of Corruption, (3) Open Government, (4) Fundamental Rights, (5) Order and Security, (6) Regulatory Enforcement, (7) Civil Justice, and (8) Criminal Justice.

Page 15: CSE-PRELIMS 2018 • PAPER-1 · 2 | Detailed of Solutions of CSE-Prelims:2018 • Paper-1 Analysis of 2018 Prelims: 1. The question paper was difficult as compared to the last year’s

Detailed of Solutions of CSE-Prelims:2018 • Paper-1 | 15

26

Which of the following led to the introduction of EnglishEducation in India?

1. Charter Act of 18132. General Committee of Public Instruction, 18233. Orientalist and Anglicist Controversy

Select the correct answer using the code given below :(a) 1 and 2 only(b) 2 only(c) 1 and 3 only(d) 1, 2 and3

d Education

All the three given laws/events facilitatedintroduction of English education in India.Under Charter Act of 1813, official moneyof rupees one lac was allotted to expandthe education of the Indians, whichcompelled East India Company to acceptresponsibility for the education of theIndian people. As a result, from 1813 to1857, the company opened many schoolsand colleges under their control which laidthe foundation of the English system ofeducation in India.General Committee of Public Instruction,1823 under chairmanship of LordMacauley took a decisive stand againstthe native culture and learning, nativeknowledge and languages like Sanskrit,Arabic and Persian and asserted on theeducation of Western Science throughEnglish language.All of those groups who may be called‘Anglicists’, in general believed thatIndians were in a backward stage andWestern education given through Englishlanguage alone was the remedy, whichhelped in spread of English Education inIndia.

http://www.kkhsou.in/main/education/charter_act.htmlhttp://www.nios.ac.in/media/documents/dled/Block1_501.pdf

Difficult

Test 1, Question no. 64Which of the following is/are related todevelopment of education in British India? 1. Charter act of 1813 2. Lord Macaulay’s Minute (1835) 3. Wood’s Despatch (1854) Select the correct answer using the code givenbelow: (a) 1 and 2 only (b) 1 and 3 only (c) 2 and 3 only (d) 1, 2 and 3 Solution: Statement 1 is correct: Charter Act 1813directed the company to sanction one lakhrupees annually for promoting knowledge ofModern Sciences. Statement 2 is correct: LordMacaulay’s Famous Minute conclude that limitedgovernment resources were to be devoted toteaching of western sciences. Statement 3 iscorrect: In 1854 , Charles wood prepared adespatch on an educational system in India alsoknown as Magna Carta of English education inIndia.

27

Which one of the following is an ‘artificial lake‘?

(a) Kodaikanal (Tamil Nadu)(b) Kolleru (Andhra Pradesh)(c) Nainital (Uttarakhand)(d) Renuka (Himachal Pradesh)

a Water Bodies

Kodaikanal Lake was created in 1863 bySir Hendry Levinge. He was previouslythe Collector of Madurai, who retired andsettled in Kodaikanal.

http://www.environment.tn.nic.in/doc/Kodaikanal%20Lak1-22.07.09.doc

Medium

Page 16: CSE-PRELIMS 2018 • PAPER-1 · 2 | Detailed of Solutions of CSE-Prelims:2018 • Paper-1 Analysis of 2018 Prelims: 1. The question paper was difficult as compared to the last year’s

16 | Detailed of Solutions of CSE-Prelims:2018 • Paper-1

28

With reference to Pradhan Mantri Kaushal Vikas Yojana,consider the following statements :

1. It is the flagship scheme of the Ministry of Labour andEmployment.2. It, among other things, will also impart training in softskills, entrepreneurship, financial and digital literacy.3. It aims to align the competencies of the unregulatedworkforce of the country to the National SkillQualification Framework.Which of the statements given above is/are correct?

(a) 1 and 3 only(b) 2 only(c) 2 and 3 only(d) 1, 2 and 3

c Schemes and Policies

Statement 1 is not correct: PradhanMantri Kaushal Vikas Yojana (PMKVY) isthe flagship scheme of the Ministry of SkillDevelopment & Entrepreneurship(MSDE).Statement 2 is correct: Apart fromproviding training according to theNational Skills Qualification Framework(NSQF), TCs shall also impart training inSoft Skills, Entrepreneurship, Financialand Digital Literacy. Statement 3 is correct: A keycomponent of scheme, Recognition ofPrior Learing (RPL) aims to align thecompetencies of the unregulatedworkforce of the country to the NSQF.

http://www.skilldevelopment.gov.in/pmkvy.html Easy

Test 5, Question 75:Consider the following statements regarding PradhanMantri Kaushal Vikas Yojana (PMKVY), the flagshipscheme for skill training of youth: 1. Skill training would be done based on the NationalSkill Qualification Framework (NSQF) and industry ledstandards. 2. It will be implemented by the new Ministry of SkillDevelopment and Entrepreneurship through theNational Skill Development Corporation (NSDC). Which of the statements give above is/are correct? (a) 1 only (b) 2 only (c) Both 1 and 2 (d) Neither 1 nor 2Solution: Pradhan Mantri Kaushal Vikas Yojana (PMKVY) is theflagship scheme of the Ministry of Skill Development &Entrepreneurship (MSDE). The objective of this SkillCertification Scheme is to enable a large number ofIndian youth to take up industry-relevant skill trainingthat will help them in securing a better livelihood.Individuals with prior learning experience or skills willalso be assessed and certified under Recognition ofPrior Learning (RPL). Under this Scheme, Training andAssessment fees are completely paid by theGovernment. Statement 1 is correct: Skill training would be donebased on the National Skill Qualification Framework(NSQF) and industry led standards. The National SkillsQualifications Framework (NSQF) is a competency-based framework that organizes all qualificationsaccording to a series of levels of knowledge, skills andaptitude. These levels, graded from one to ten, aredefined in terms of learning outcomes which the learnermust possess regardless of whether they are obtainedthrough formal, non-formal or informal learning. NSQF inIndia was notified on 27th December 2013. All otherframeworks, including the NVEQF (National VocationalEducational Qualification Framework) released by theMinistry of HRD, stand superseded by the NSQF. Statement 2 is correct: This is the flagship scheme forskill training of youth to be implemented by the newMinistry of Skill Development and Entrepreneurshipthrough the National Skill Development Corporation(NSDC). The scheme will cover 10 million youth duringthe period 2016-2020.

29

In 1920, which of the following changed its name to“Swarajya Sabha” ?

(a) All India Home Rule League(b) Hindu Mahasabha(c) South Indian Liberal Federation(d) The Servants of India Society

a Freedom Struggle

Gandhi secured the presidency of theHome Rule League when he was invitedto chair a meeting in Bombay on 3rdOctober 1920. There he changed thename of the association to SwarajyaSabha, not 'Home Rule' anymore, but anassociation for independence.

Gandhi's Passion: The Life and Legacy of Mahatma GandhiBy Stanley Wolper Pg 110Gandhi and Nationalism: The Path to Indian IndependenceBy Simone Panter-Brick pg 32

Difficult

Page 17: CSE-PRELIMS 2018 • PAPER-1 · 2 | Detailed of Solutions of CSE-Prelims:2018 • Paper-1 Analysis of 2018 Prelims: 1. The question paper was difficult as compared to the last year’s

Detailed of Solutions of CSE-Prelims:2018 • Paper-1 | 17

30

Which among the following events happened earliest?

(a) Swami Dayanand established Arya Samaj.(b) Dinabandhu Mitra wrote Neeldarpan(c) Bankim Chandra Chattopadhyay wrote Anandmath.(d) Satyendranath Tagore became the first Indian tosucceed in the Indian Civil Services Examination.

b Personalities of Freedom Struggle

Swami Dayanand Saraswati establishedArya Samaj in 1875.

In 1860, Dinabandhu Mitra, an inspectorof the post office department, published aplay from Dhaka titled Neel Darpan.

Bankim Chandra Chattopadhyay wroteAnandmath in 1882.

Satyendranath Tagore became the firstIndian to succeed in the Indian CivilServices Examination in 1863.

BL Grover Page no. 489, 278

http://www.open.ac.uk/researchprojects/makingbritain/content/satyendranath-tagore

Difficult

Test 14, Explanation 65:The Arya Samaj founded by Swami DayanandSaraswati in 1875 undertook the task ofreforming Hindu religion in North India.Test 14, Explanation 4:Dinabandhu Mitra published in September 1860a play in Bengali called Neel Darpan (literally,‘blue mirror’), which depicted the atrocities of theindigo planters in the boldest possible colour.Test 28, Explanation 44:Bankimchandra Chatterjii ‘s novel AnandaMath(1882), set against the background of theSannyasi rebellion in Bengali, contained thefamous nationalist song 'Bande Mataram'.

31

Which of the following is/are the possible consequence/sof heavy sand mining in riverbeds ?

1. Decreased salinity in the river2. Pollution of groundwater3. Lowering of the water-table

Select the correct answer using the code given below :(a) 1 only(b) 2 and 3 only(c) 1 and 3 only(d) 1, 2 and 3

b Water Pollution

In NCR, the illegal sand mining mostlyoccurs along the Yamuna riverbed in thecities of Delhi, Noida, Haryana and theAravalli ranges. This indiscriminate sandmining in the river beds of Yamuna &Ghaggar has threatened the biodiversity,destroyed riverine vegetation, causederosion, polluted water sources etc.Over the years, extensive damage hasbeen caused to the ecosystem of theserivers, has weakenedthe river beds,spelled disaster for the conservation ofmany bird species, increased salinityof water in the rivers etc.Statement(1) is not correct: Heavy sandmining in riverbed would lead toincreased salinity in the river. Depletionof sand in the streambed causes thedeepening of rivers and estuaries, andthe enlargement of river mouths andcoastal inlets. It leads to saline-waterintrusion. Statement(2) is correct: Sand miningwould lead to pollution of groundwater.Statement(3) is correct: Heavy sandmining would lead to lowering of watertable.

http://www.nluassam.ac.in/data3/lexterraissue23/Article%202.pdfhttp://envfor.nic.in/sites/default/files/Final%20Sustainable%20Sand%20Mining%20Management%20Guidelines%202016.pdfhttp://shodhganga.inflibnet.ac.in/bitstream/10603/8987/12/12_chapter%208.pdfhttp://www.nluassam.ac.in/data3/lexterraissue23/Article%202.pdf

Medium

Page 18: CSE-PRELIMS 2018 • PAPER-1 · 2 | Detailed of Solutions of CSE-Prelims:2018 • Paper-1 Analysis of 2018 Prelims: 1. The question paper was difficult as compared to the last year’s

18 | Detailed of Solutions of CSE-Prelims:2018 • Paper-1

32

With reference to agricultural soils, consider thefollowing statements :

1. A high content of organic matter in soil drasticallyreduces its water holding capacity.2. Soil does not play any role in the sulphur cycle.3. Irrigation over a period of time can contribute to thesalinization of some agricultural lands.

Which of the statements given above is/are correct ?(a) 1 and 2 only(b) 3 only(c) 1 and 3 only(d) 1, 2 and 3

b Soil Pollution

Statement 1 is not correct: Plantresidues that cover the soil surfaceprotect the soil from sealing and crustingby raindrop impact, thereby enhancingrainwater infiltration and reducing runoff.Increased organic matter contributesindirectly to soil porosity (via increasedsoil faunal activity).Statement 2 is not correct: Sulfur cycleis circulation of sulfur in various formsthrough nature. Sulfur occurs in all livingmatter as a component of certain aminoacids. It is abundant in the soil in proteinsand, through a series of microbialtransformations, ends up as sulfatesusable by plants.

Statement 3 is correct: Secondary salinization occurs when significantamounts of water areprovided by irrigation, with no adequateprovision of drainage for theleaching and removal of salts, resulting inthe soils becoming salty andunproductive.

Statement 1: http://www.fao.org/docrep/009/a0100e/a0100e08.htmStatement 2: https://www.sciencedirect.com/topics/agricultural-and-biological-sciences/sulfur-cyclestatement 3: http://www.fao.org/tempref/agl/agll/docs/salinity_brochure_eng.pdf

Medium

Test 4, explanation 7:The movement of nutrient elements through thevarious components of an ecosystem is callednutrient cycling. Another name of nutrient cyclingis biogeochemical cycles. Nutrient cycles are oftwo types: (a) gaseous and (b) sedimentary.The reservoir for gaseous type of nutrientcycle (e.g., nitrogen, carbon cycle) exists inthe atmosphere and for the sedimentary cycle(e.g., sulphur and phosphorus cycle), thereservoir is located in Earth’s crust.Environmental factors, e.g., soil, moisture, pH,temperature, etc., regulate the rate of release ofnutrients into the atmosphere. The function of thereservoir is to meet with the deficit which occursdue to imbalance in the rate of influx and efflux. Test 30, Explanation 8:Saline soils contain a larger proportion ofsodium, potassium and magnesium, and thus,they are infertile, and do not support anyvegetative growth. They have more salts, largelybecause of dry climate and poor drainage. Theyoccur in arid and semi-arid regions, and inwaterlogged and swampy areas. Their structureranges from sandy to loamy. They lack innitrogen and calcium.

33

The Partnership for Action on Green Economy (PAGE),a UN mechanim to assist countries transition towardsgreener and more inclusive economies, emerged at

(a) The Earth Summit on Sustainable Development2002, Johannesburg(b) The United Nations Conference on SustainableDevelopment 2012, Rio de Janeiro(c) The United Nations Framework Convention onClimate Change 2015, Paris(d) The World Sustainable Development Summit 2016,New Delhi

bInternational

Institutions/Partnerships

The Partnership for Action on GreenEconomy (PAGE) was launched in 2013as a response to the call at Rio+20 tosupport those countries wishing toembark on greener and more inclusivegrowth trajectories.

PAGE seeks to put sustainability at theheart of economic policies and practicesto advance the 2030 Agenda forSustainable Development and supportsnations and regions in reframingeconomic policies and practices aroundsustainability to foster economic growth,create income and jobs, reduce povertyand inequality, and strengthen theecological foundations of their economies.

http://www.un-page.org/about/who-are-we

Medium

Page 19: CSE-PRELIMS 2018 • PAPER-1 · 2 | Detailed of Solutions of CSE-Prelims:2018 • Paper-1 Analysis of 2018 Prelims: 1. The question paper was difficult as compared to the last year’s

Detailed of Solutions of CSE-Prelims:2018 • Paper-1 | 19

34

“3D printing” has applications in which of the following?

1. Preparation of confectionery items2. Manufacture of bionic ears3. Automotive industry4. Reconstructive surgeries5. Data processing technologies

Select the correct answer using the code given below :(a) 1, 3 and 4 only(b) 2, 3 and 5 only(c) 1 and 4 only(d) 1, 2, 3, 4 and 5

d Information Technology

3D printing technologies can be used todesign and build everything fromautomobile parts to medical devices usingadditive manufacturing technology.

Because additive manufacturing typicallycreates products layer by layer,manufacturers will also have theopportunity, and may feel the need, tomonitor production in entirely new ways.Closely monitoring products as they areformed could yield terabytes of data. CIOswill need to determine what data is usefuland should be maintained, and whatshould be discarded (and when). Theability to maintain and analyze valuabledata may become increasingly importantas 3D printing technologies are deployedin the production of engine parts, medicaldevices, and other items upon whichhuman safety depends.

https://www.forbes.com/sites/forbestechcouncil/2018/02/06/11-tech-pros-share-their-favorite-applications-of-3d-printing-technology/2/#39ee704f5991

http://deloitte.wsj.com/cio/2013/06/25/3d-printing-data-data-everywhere/

Medium

35

Consider the following statements:

1. The Barren Island volcano is an active volcanolocated in the Indian territory.2. Barren Island lies about 140 km east of GreatNicobar.3. The last time the Barren Island volcano erupted wasin 1991 and it has remained inactive since then.

Which of the statements given above is/are correct ?(a) 1 only(b) 2 and 3(c) 3 only(d) 1 and 3

a Indian Geography

Statement 1 is correct: The BarrenIsland volcano is an active volcanolocated in the Indian territory.Statement 2 is not correct: The BarrenIsland, about 140 km from Port Blair, is atourist destination and surrounded bywaters ideal for scuba diving and is hometo a wide variety of aquatic life. Port Blairlies in South Andaman. Barren Islanddoes not lie to the east of Great Nicobar.It lies approximately in between MiddleAndaman and South Andaman.Statement 3 is not correct: Barren islanderupted recently in 2017.Thus, option a is correct.

http://www.thehindu.com/news/national/the-barren-islandvolcano-erupts-again/article17369862.ece

Medium

Page 20: CSE-PRELIMS 2018 • PAPER-1 · 2 | Detailed of Solutions of CSE-Prelims:2018 • Paper-1 Analysis of 2018 Prelims: 1. The question paper was difficult as compared to the last year’s

20 | Detailed of Solutions of CSE-Prelims:2018 • Paper-1

36

Why is a plant called Prosopis juliflora often mentionedin news?

(a) Its extract is widely used in cosmetics.(b) It tends to reduce the biodiversity in the area in whichit grows.(c) Its extract is used in the synthesis of pesticides.(d) None of the above

b Ecology

Prosopis juliflora, commonly known asVilayati Keekar or Seemai karuvelam isan invasive species. It tends to reduce thebiodiversity in the area in which it grows.

http://indianexpress.com/article/explained/delhi-government-vilayati-kikar-removal-order-environmental-impact-5194797/

Difficult

Test 5, question no. 100Consider the following statements about the‘Seemai Karuvelam’ tree which was recently innews: 1. It is tolerant to arid conditions and saline soil. 2. It is native to Tamil Nadu and adjoiningregions. Which of the statements given aboveis/are correct? (a) 1 only (b) 2 only (c) Both 1 and 2 (d) Neither 1 nor 2 Statement 1 is correct: ‘Prosopis juliflora’,(seemai karuvel), once touted as a saviour of thedrought-prone areas in the southern districts ofTamil Nadu, has now become a threat to theenvironmental system. It is a fast growing treevariety and tolerant to arid conditions and salinesoil Statement 2 is not correct: The evergreenspecies, ‘Prosopis juliflora’, (seemai karuvel), isnative to the South and Central America and theCaribbean. Prosopis was introduced in Indiaduring the 1870s to meet the fuel wood demandand in Tamil Nadu the 1960s, particularly in thecomposite Ramanathapuram and Tirunelvelidistricts.

37

Consider the following statements :

1. Most of the world’s coral reefs are in tropical waters.2. More than one-third of the world’s coral reefs arelocated in the territories of Australia, Indonesia andPhilippines.3. Coral reefs host far more number of animal phyla thanthose hosted by tropical rainforests.

Which of the statements given above is/are correct ?(a) 1 and 2 only(b) 3 only(c) 1 and 3 only(d) 1, 2 and 3

d Aquatic Ecosystems

Statement 1 is correct: Most of theworld's coral reefs are in tropical waters.Statement 2 is correct: More than one-third of the world's coral reefs are locatedin the territories of Australia (17.22%),Indonesia (17.95%) and Philippines(8.81%).Statement 3 is correct: Theseecosystems are packed with the highestdensities of animals to be found anywhere on the planet.

http://coral.unep.ch/atlaspr.htmhttp://fnad.org/Documentos/worldatlasofcora01spal.pdf

Difficult

38

“Momentum for Change: Climate Neutral Now” is aninitiative launched by

(a) The Intergovernmental panel on Climate Change(b) The UNEP Secretariat(c) The UNFCCC Secretariat(d) The World Meteorological Organization

cInternational

Institutions/Partnerships

The UNFCCC secretariat launched itsClimate Neutral Now initiative in 2015.

https://unfccc.int/climate-action/momentum-for-change/climate-neutral-now

Medium

Page 21: CSE-PRELIMS 2018 • PAPER-1 · 2 | Detailed of Solutions of CSE-Prelims:2018 • Paper-1 Analysis of 2018 Prelims: 1. The question paper was difficult as compared to the last year’s

Detailed of Solutions of CSE-Prelims:2018 • Paper-1 | 21

39

With reference to educational institutions during colonialrule in India, consider the following pairs:

Institution Founder1. Sanskrit College at Benaras William Jones2. Calcutta Madarsa Warren Hastings3. Fort William College Arthur Wellesley

Which of the pairs given above is/are correct ?(a) 1 and 2(b) 2 only(c) 1 and 3(d) 3 only

b Modern Indian History

Calcutta Madrassa was founded byWarren Hastings(1781), the AsiaticSociety of Bengal founded by WilliamJones (1784) and the Sanskrit Collegeat Benaras founded by Jonathan Duncan(1794). There was a strong urge tomake local British administratorsfamiliar with Indian culture and tradition.Fort William College founded by LordWellesley in 1801 to train the youngBritish recruits to the civil service in Indiawas meant to serve primarily thispurpose. Note that here the name given isArthur Wellesly, which is wrong asRichard Wellesley was Governor-General (1798-1805).

http://www.nios.ac.in/media/documents/SrSec315NEW/History_Module3.pdf

http://ncert.nic.in/ncerts/l/hess102.pdf

Difficult

Prelims Test Series- Test Number 15

Q.25 Consider the following pairs:Institution Founder1. Calcutta Madarsa : Warren Hastings2. Fort William College : Wellesley3. Asiatic Society of Bengal : William JonesWhich of the pairs given above is/are correctlymatched?(a) 1 and 2 only (b) 3 only(c) 2 and 3 only (d) 1, 2 and 325. (d)The early British administrators in India likeWarren Hastings, William Jones, JonathanDuncan and others glorified India’s ancient past.These scholars and administrators were calledOrientalists. Important institutions that came tobe identified with their efforts were the CalcuttaMadarsas founded by Warren Hastings (1781),the Asiatic Society of Bengal founded by WilliamJones (1784), the Sanskrit College at Banarasfounded by Jonathan Duncan (1794) and the Fort William College founded by Wellesley (1800).

40

Consider the following pairs :

Regions sometimes mentioned in newsCountry1. Catalonia Spain2. Crimea Hungary3. Mindanao Philippines4. Oromia Nigeria

Which of the pairs given, above are correctly matched ?(a) 1, 2 and 3(b) 3 and 4 only(c) 1 and 3 only(d) 2 and 4 only

c Mapping

Catalonia recently voted to seperate fromSpain.Crimea was in news because of Russianannexation of Crimea from Ukraine. Forthis purpose a referendum was conductedin 2014.Mindanao is in Phlippines.Oromia is in Nigeria.

http://www.thehindu.com/opinion/lead/why-russia-needs-crimea/article5792952.ecehttp://www.thehindu.com/news/international/all-you-need-to-know-about-catalonias-independence-referendum/article18984755.ece

Easy

Prelims Test series-Test 23Q.59 Recently, ‘Catalonia’ has been in news. It isassociated with which one of the followingcountries?(a) Spain(b) Netherlands(c) Norway(d) Britain59. (a)Catalonia’s parliament passed a motion todeclare independence from Spain in October2017. It is situated in the south-east region of theSpain and open up in Mediterranean sea,touching France in North.

Magazine (Current affairs Made Easy)Catalonia is mentioned in November, 2017 edition (Page 58)

Page 22: CSE-PRELIMS 2018 • PAPER-1 · 2 | Detailed of Solutions of CSE-Prelims:2018 • Paper-1 Analysis of 2018 Prelims: 1. The question paper was difficult as compared to the last year’s

22 | Detailed of Solutions of CSE-Prelims:2018 • Paper-1

41

With reference to the Indian Regional NavigationSatellite System (IRNSS), consider the followingstatements :

1. IRNSS has three satellites in geostationary and foursatellites in geosynchronous orbits.2. IRNSS covers entire India and about 5500 sq. kmbeyond its borders.3. India will have its own satellite navigation system withfull global coverage by the middle of 2019.

Which of the statements given above is/are correct?(a) 1 only(b) 1 and 2 only(c) 2 and 3 only(d) None

a Space Technnology

Statement 1 is correct: Three satellitesare located in suitable orbital slots in thegeostationary orbit and the remaining fourare located in geosynchronous orbits withthe required inclination and equatorialcrossings in two different planes. Statement 2 is not correct: It isdesigned to provide accurate positioninformation service to users in India aswell as the region extending up to 1500km from its boundary, which is its primaryservice area.Statement 3 is not correct: Thecoverage of IRNSS is not global butregional.

https://www.isro.gov.in/irnss-programmehttp://www.newindianexpress.com/cities/bengaluru/2018/apr/13/navic-to-cover-indian-mainland-and-1500-km-beyond-countrys-borders-1800886.html

Medium

Magazine (Current affairs Made Easy)May 2018 edition (Page 111)

42

Consider the following phenomena :

1. Light is affected by gravity.2. The Universe is constantly expanding.3. Matter warps its surrounding space-time.

Which of the above is/are the prediction/predictions ofAlbert Einstein’s General Theory of Relativity, oftendiscussed in media?(a) 1 and 2 only(b) 3 only(c) 1 and 3 only(d) 1, 2 and 3

d General Science

Statement 1 is correct: Light is affectedby gravity. Statement 2 is correct: Astronomersusing NASA’s Hubble Space Telescopehave discovered that the universe isexpanding 5 percent to 9 percent fasterthan expected. For how fast the universeexpands with time, is called the Hubbleconstant.Statement 3 is correct: Almost all thewarpage in our solar system is produceddirectly by matter — the Earth’s matter,the Sun’s matter, the matter of the otherplanets.

Statement 1: http://curious.astro.cornell.edu/about-us/140-physics/the-theory-of-relativity/general-relativity/1021-if-photons-have-zero-mass-why-do-they-feel-the-effects-of-gravity-advancedStatement 2: https://www.nasa.gov/feature/goddard/2016/nasa-s-hubble-finds-universe-is-expanding-faster-than-expectedhttps://www.its.caltech.edu/~kip/index.html/PubScans/VI-47.pdfStatement 3 : https://www.its.caltech.edu/~kip/index.html/PubScans/VI-47.pdf

Medium

43

With reference to the Genetically Modified mustard (GMmustard) developed in India, consider the followingstatements :

1. GM mustard has the genes of a soil bacterium thatgive the plant the property of pest-resistance to a widevariety of pests.2. GM mustard has the genes that allow the plant cross-pollination and hybridization.3. GM mustard has been developed jointly by the IARIand Punjab AgricuItal University.

Which of the statements given above is/are correct?(a)1 and 3 only(b) 2 only(c)2 and 3(d)1, 2 and 3

b Biotechnology

GM mustard has been developed by ateam of scientists at Delhi University ledby former vice-chancellor Deepak Pentalunder a government-funded project. Inessence, it uses a system of genes fromsoil bacterium that makes mustard —generally a self pollinating plant — bettersuited to hybridisation than currentmethods.

http://envfor.nic.in/sites/default/files/press-releases/Final_FAQs_GMM_website1.pdf http://www.ifpri.org/blog/let-dhara-flow-gm-mustard-pro-farmer-and-pro-sciencehttp://www.thehindu.com/news/national/GM-mustard-moves-closer-to-approval/article14589131.ece

Medium

Newslog (May_15_2018)Statement 3 is incorrect. The correct statement was mentioned in Newslog (May_15_2018)” “Dhara Mustard Hybrid (DMH -11), the transgenic mustard in question, had been developed by a team of scientists at Delhi University, under a government-funded project’.

Page 23: CSE-PRELIMS 2018 • PAPER-1 · 2 | Detailed of Solutions of CSE-Prelims:2018 • Paper-1 Analysis of 2018 Prelims: 1. The question paper was difficult as compared to the last year’s

Detailed of Solutions of CSE-Prelims:2018 • Paper-1 | 23

44

Consider the following pairs :

Terms sometimes seen in news Context /Topic1. Belle II experiment Artificial Intelligence2. Blockchain technology Digital/ Cryptocurrency3. CRISPR — Cas9 Particle Physics

Which of the pairs given above is/are correctlymatched?(a) 1 and 3 only(b) 2 only(c) 2 and 3 only(d) 1, 2 and 3

b Science and Technology

Pair 1 is not correct: The Belle IIexperiment is a particle physicsexperiment designed to study theproperties of B mesons Pair 2 is correct:Blockchain technology, usesdecentralised ledger whichhas application in the crypto-currency Pair 3 is not correct: CRISPR-Cas9 isgenome editing system.It can remove,add or alter specific DNA sequences inthegenome of higher organisms

https://www.sciencedirect.com/science/article/pii/S240560141500108Xhttps://www.belle2.org/http://www.idrbt.ac.in/assets/publications/Best%20Practices/BCT.pdfhttps://ghr.nlm.nih.gov/primer/genomicresearch/genomeediting

Easy

Prelims Test Series : Test 25Q.11 Consider the following statements regarding‘CRISPR-Cas9’, which was in news recently withreference to genome editing.1. CRISPR-Cas9 was adapted from a naturallyoccurring genome editingsystem in bacteria.2. The Cas9 is an enzyme that cuts foreign DNA.Which of the statements given above is/are correct?(a) 1 only (b) 2 only(c) Both 1 and 2 (d) Neither 1 nor 211. (c)Both the given statements are correct: CRISPRs arespecialized stretches of DNA. The protein Cas9 (or‘CRISPR-associated’) is an enzyme thatacts like a pair of molecular scissors, capable ofcutting strands of DNA.CRISPR technology was adapted from the naturaldefense mechanisms of bacteria and archaea (thedomain of single-celled microorganisms). Theseorganisms use CRISPR-derived RNA and variousCas proteins, including Cas9, to foil attacks by virusesand other foreign bodies. They do so primarily bychopping up and destroying the DNA of a foreigninvader. When these components are transferred intoother, more complex, organisms, it allows for themanipulation of genes, or ‘editing’. ‘CRISPR’ standsfor clusters of regularly interspaced short palindromicrepeats.” It is a specialized region of DNA with twodistinct characteristics: the presence of nucleotiderepeats and spacers. Pelims Test Series : Test 20Q.15 Consider the following statements about‘Blockchain Technology’ recently in news inconnection with cryptocurrencies:1. All cryptocurrencies are based on blockchaintechnology.2. It is a distributed digital public ledger that recordsevery transaction.3. Venezuela has launched world’s first oil backedsovereign cryptocurrency named ‘Petro’.Which of the statements given above is/are correct?(a) 1 and 2 only (b) 1 and 3 only(c) 2 and 3 only (d) 1, 2 and 3

Magazine (Current affairs Made Easy)Blockchain technology is mentioned in October, 2017 edition (Page 39)CRISPR-Cas9 is mentioned in July-August-September, 2017 edition (Page 150)

45

Which of the following statements best describes“carbon fertilization” ?

(a) Increased plant growth due to increasedconcentration of carbon dioxide in the atmosphere(b) Increased temperature of Earth due to increasedconcentration of carbon dioxide in the atmosphere(c) Increased acidity of oceans as a result of increasedconcentration of carbon dioxide in the atmosphere(d) Adaptation of all living beings on Earth to the climatechange brought about by the increased concentration ofcarbon dioxide in the atmosphere.

a Ecology

The carbon fertilization effect (CFE) is inprinciple simple: the larger amount ofcarbon dioxide in the atmosphere that hasresulted from rising anthropogenicemissions should help the growth ofplants, which use carbon dioxide duringphotosynthesis. The effect ought toincrease crop yields – and that is somegood news for farmers, amid theoverwhelmingly gloomy forecasts forother aspects of climate change.

https://www.nasa.gov/feature/goddard/2016/carbon-dioxide-fertilization-greening-earth

http://environmentalresearchweb.org/cws/article/news/54347

Easy

Page 24: CSE-PRELIMS 2018 • PAPER-1 · 2 | Detailed of Solutions of CSE-Prelims:2018 • Paper-1 Analysis of 2018 Prelims: 1. The question paper was difficult as compared to the last year’s

24 | Detailed of Solutions of CSE-Prelims:2018 • Paper-1

46

When the alarm of your smartphone rings in themorning, you wake up and tap it to stop the alarm whichcauses your geyser to be switched on automatically. Thesmart mirror in your bathroom shows the day’s weatherand also indicates the level of water in your overheadtank. After you take some groceries from yourrefrigerator for making breakfast, it recognises theshortage of stock in it and places an order for the supplyof fresh grocery items. When you step’ out of your houseand lock the door, all lights, fans, geysers and ACmachines get switched off automatically. On your way tooffice, your car warns you about traffic congestion aheadand suggests an alternative route, and if you are late fora meeting, it sends a message to your office accordingly.

In the context of emerging communication technologies,which one of the following terms best applies to theabove scenario?(a) Border Gateway Protocol(b) Internet of Things(c) Internet Protocol(d) Virtual Private Network

b Information Technology

The internet of things (IoT) is a computingconcept that describes the idea ofeveryday physical objects beingconnected to the internet and being ableto identify themselves to other devices.The term is closely identified with RFID asthe method of communication, although italso may include other sensortechnologies, wireless technologies or QRcodes.The IoT is significant because an objectthat can represent itself digitally becomessomething greater than the object byitself. No longer does the object relate justto its user, but it is now connected tosurrounding objects and database data.When many objects act in unison, theyare known as having "ambientintelligence."

https://www.cisco.com/c/dam/en_us/solutions/trends/iot/introduction_to_IoT_november.pdfhttps://www.gsma.com/iot/wp-content/uploads/2014/08/cl_iot_wp_07_14.pdfhttps://www.techopedia.com/definition/28247/internet-of-things-iot

Easy

47

With reference to solar power production in India,consider the following statements :

1. India is the third largest in the world in themanufacture of silicon wafers used in photovoltaic units.2. The solar power tariffs are determined by the SolarEnergy Corporation of India.

Which of the statements given above is/are correct ?(a) 1 only(b) 2 only(c) Both 1 and 2(d) Neither 1 nor 2

d Energy

Both the given statements are notcorrect: . India is not among the top threeprducers in the world in the manufactureof silicon wafers used in photovoltaicunits. The top producers are- China,Russia, USA, Norway, France, Brazil,South Africa, Iceland etc. Solar Energy Corporation of India Ltd.(SECI) is a CPSU under theadministrative control of the Ministry ofNew and Renewable Energy (MNRE).The company is responsible forimplementation of a number of schemesof MNRE, major ones being the VGFschemes for large-scale grid-connectedprojects under JNNSM, solar parkscheme and grid-connected solar rooftopscheme. SECI is not involved indetermining solar power tariffs.

http://seci.gov.in/upload/uploadfiles/files/FAQ.pdf

https://www.statista.com/statistics/268108/world-silicon-production-by-country/

Difficult

48

The staple commodities of export by the English EastIndia Company from Bengal in the middle of the 18thcentury were

(a) Raw cotton, oil-seeds and opium(b) Sugar, salt, zinc and lead(c) Copper, silver, gold, spices and tea(d) Cotton, silk, saltpetre and opium

d Modern Indian History

India of the 18th century failed to makeprogress economically, socially orculturally at an adequate pace. India alsoexported raw silk and silk fabrics,hardware, indigo, saltpetre, opium, rice,wheat, sugar, pepperand other spices, precious stones anddrugs.

OLD NCERT-INDIAN MODERN HISTORY Difficult

Page 25: CSE-PRELIMS 2018 • PAPER-1 · 2 | Detailed of Solutions of CSE-Prelims:2018 • Paper-1 Analysis of 2018 Prelims: 1. The question paper was difficult as compared to the last year’s

Detailed of Solutions of CSE-Prelims:2018 • Paper-1 | 25

49

Which one of the following is a very significant aspect ofthe Champaran Satyagraha ?

(a) Active all-India participation of lawyers, students andwomen in the National Movement(b) Active involvement of Dalit and Tribal communities ofIndia in the National Movement(c) Joining of peasant unrest to India’s NationalMovement(d) Drastic decrease in the cultivation of plantation cropsand commercial crops

c Modern Indian History

Gandhi's first great experiment in satyagrahacame in 1917 in Champaran, a district in Bihar.ThePeasantry on the indigo plantations in thedistrict was excessively oppressed by theEuropeanplanters. They were compelled to grew indigoon at least 3/20th of their land and to sell it atprices fixed by the plantern. Similar conditionshad prevailed earlier in Bengal, but as a resultofa major uprising during 1859-61 the peasantsthere had won the freedom from the indigoplanters.The Champaran satyagraha was the firstintersection of peasant unrest and the nationalmovement.

http://indianexpress.com/article/opinion/columns/celebrating-champaran-satyagraha-of-1917-british-rule-indian-freedom-movement-4628343/

Medium

50

Who among the following were the founders of the “HindMazdoor Sabha” established in 1948 ?

(a) B. Krishna Pillai, E.M.S. Namboodiripad and K.C.George(b) Jayaprakash Narayan, De Dayal Upadhyay and M.N.Roy(c) C.P. Ramaswamy Iyer, K. Kamaraj andVeeresalingam Pantulu(d) Ashok Mehta, T.S. Ramanujam and G.G. Mehta

d Personalities of Freedom Struggle

HMS was founded in Calcutta in 1948. TThe Founding Conference elected Com.R.S. Ruikar as the first President, Com.Ashok Mehta as the General Secretaryand Com. G.G. Mehta and V.S. Mathur asSecretaries. The members of the WorkingCommittee included veteran leaders like -Jayaprakash Narayan, V.G. Dalvi, Ms.Aruna Asaf Ali, V.B. Karnik, Dinkar Desai,N.V. Phadke, M.V. Donde, RajaniMukherjee, Haren Ghosh, Anthoni Pillai,P.S. Chinnadurai, Peter Alwares, A.M.Williams, Munshi Ahmed Din, VinayakKulkarni, Nibran Ch. Bora and BasawanSingh.

http://www.hindmazdoorsabha.com/about-hms.php Difficult

51

With reference to the religious practices in India, the“Sthanakvasi” sect belongs to

(a) Buddhism(b) Jainism(c) Vaishnavism(d) Shaivism

b Culture

Sthanakvasi arose as reformer of an older reforming sect, viz., the Lonka sect ofJainism. This Lonka sect was founded inabout 1474 A.D. by Lonkashaha, a richand well-read merchant of Ahmedabad.The main principle of this sect was not topractice idol-worship.

http://indianexpress.com/article/india/india-others/the-rite-death-santhara-jain/

Difficult

Page 26: CSE-PRELIMS 2018 • PAPER-1 · 2 | Detailed of Solutions of CSE-Prelims:2018 • Paper-1 Analysis of 2018 Prelims: 1. The question paper was difficult as compared to the last year’s

26 | Detailed of Solutions of CSE-Prelims:2018 • Paper-1

52

With reference to the cultural history of India, considerthe following statements :

1. White marble was used in making Buland Darwazaand Khankah at Fatehpur Sikri.2. Red sandstone and marble were used in making BaraImambara and Rumi Darwaza at Lucknow.

Which of the statements given above is/are correct?(a) 1 only(b) 2 only(c) Both 1 and 2(d) Neither 1 nor 2

a Architecture

Statement 1 is correct :Buland Darwazaincorporates almost all the essentialfeatures of Akbar’s architecturaltraditions: red sandstone, stone carvings,and relief by inserting white marble. White marble was used in making bothBuland Darwaza and Khankah atFatehpur Sikri.Statement 2 is not correct : Thematerial used to raise the mammothstructure of Bada Imambara was variedsuch as lakhauri brick, the uniquemasala comprising surkhi, choona, uraddaal, sheera (molasses), choone ka panior farez (a natural fevicol like substancederived from a tree). But readstone wasnot used in it.

http://www.thehindu.com/opinion/columns/buland-darwaza-and-rumi-darwaza-gateways-to-heaven/article19700177.ece

https://timesofindia.indiatimes.com/city/lucknow/What-makes-Imambara-a-green-building/articleshow/47089758.cms

Difficult

53

Which one of the following foreign travellers elaboratelydiscussed about diamonds and diamond mines of India?

(a) Francois Bernier(b) Jean-Baptiste Tavernier(c) Jean de Thevenot(d) Abbe Barthelemy Carre

b Modern Indian History

Tavernier was a diamond dealer who hadwritten on the diamond mines in theKingdom of Golconda.

http://ncert.nic.in/ncerts/l/gess106.pdf http://shodhganga.inflibnet.ac.in/bitstream/10603/823/9/09_chapter%203.pdf

http://thefrenchblue.com/timeline2.htm

Difficult

54

With reference to Indian history, who among thefollowing is a future Buddha, yet to come to save theworld?

(a) Avalokiteshvara(b) Lokesvara(c) Maitreya(d) Padmapani

c Culture Maitreya in Buddhist tradition is the futureBuddha.

https://www.britannica.com/topic/Maitreya-BuddhismAL Basham's "Wonder That Was India"ChapterReligion: cults, doctrines and metaphysics

Difficult

Page 27: CSE-PRELIMS 2018 • PAPER-1 · 2 | Detailed of Solutions of CSE-Prelims:2018 • Paper-1 Analysis of 2018 Prelims: 1. The question paper was difficult as compared to the last year’s

Detailed of Solutions of CSE-Prelims:2018 • Paper-1 | 27

55

Which one of the following statements does not apply tothe system of Subsidiary Alliance introduced by LordWellesley?

(a) To maintain a large standing army at other’s expense(b) To keep India safe from Napoleonic danger(c) To secure a fixed income for the Company(d) To establish British paramountcy over the IndianStates

c Modern Indian History

Lord Wellesley arrived as governorgeneral in 1798, with a dream of conquestand a lust for personal glory. Napoleonicinvasion of Egypt in the summer of 1798offered him a useful tool to softenLondon's resistance to expansion.Toassuage London's concerns he evolvedthe policy of 'Subsidiary Alliance', whichwould establish control over the internalaffairsof an Indian state. All those who enteredinto such an alliance with the British hadto accept certain terms and conditions:(a) The British would be responsible forprotecting their ally from external andinternal threats to their power.(b) In the territory of the ally, a Britisharmed contingent would be stationed.(c) The ally would have to provide theresources for maintaining this contingent.(d) The ally could enter into agreementswith other rulers or engage in warfare onlywith the permission of the British.

.However,the above provisions didn'tmention about a fixed income secured forthe company.Hence, statement (c) is notcorrect.

http://ncert.nic.in/ncerts/l/hess102.pdfhttp://ncert.nic.in/ncerts/l/lehs302.pdf

Plassey to partition Page No. 51

Difficult

Pelims Test Series : Test 3Q17.Consider the following statements regardingterms of Subsidiary alliance between British(paramount power) and Indian State (theprotected state)1. The protected state was forbidden to have anypolitical contact with other Indian powers without thepermission of the British.2. Indian rulers were not allowed to have theirindependent armed forces.3. The ruler of the protected state should not employEuropeans in his service without the sanction of theparamount power.4. The paramount power has rights to interfere in theinternal affairs of theprotected state.Which of the statements given above is/are correct?(a) 1, 2 and 3 only (b) 1 and 3 only(c) 2 and 4 only (d) 1, 2, 3 and 4Test 4Q27.Which of the following statements bestrepresents the ‘Policy of Ring fence’ of East IndiaCompany?1. It was introduced by Lord Wellesley who reducedthe Indian States into a position of dependence onthe British Government.2. This policy is aimed at creating buffer zones todefend the company’s frontiers.3. This policy of Subsidiary Alliance was anextension of this ring fence policy.Select the correct answer using the code givenbelow:(a) 3 only (b) 2 and 3 only(c) 1 and 2 only (d) 1, 2 and 3

56

Which one of the following statements correctlydescribes the meaning of legal tender money ?(a) The money which is tendered in courts of law todefray the fee of legal cases(b) The money which a creditor is under compulsion toaccept in settlement of his claims(c) The bank money in the form of cheques, drafts, billsof exchange, etc.(d) The metallic money in circulation in a country

b Banking

Legal tender is any official medium ofpayment recognized by law that can beused to extinguish a public or privatedebt, or meet a financial obligation. Thenational currency is legal tender inpractically every country. A creditor isobligated to accept legal tender towardrepayment of a debt.

https://www.investopedia.com/terms/l/legal-tender.aspXII: Introductory Marcoeconomics, Chapter3

Easy

Prelims Test Series : Test 2Q.86 Consider the following statementsregarding medium of exchange:1. Currency notes and coins have intrinsic value.2. Demand deposits are not legal tenders.3. Gold and silver are categorized as fiat money.Which of the statements given above is/arecorrect?(a) 1 and 2 only (b) 2 only(c) 2 and 3 only (d) 1, 2 and 3

Page 28: CSE-PRELIMS 2018 • PAPER-1 · 2 | Detailed of Solutions of CSE-Prelims:2018 • Paper-1 Analysis of 2018 Prelims: 1. The question paper was difficult as compared to the last year’s

28 | Detailed of Solutions of CSE-Prelims:2018 • Paper-1

57

If a commodity is provided free to the public by theGovernment, then

(a) the opportunity cost is zero.(b) the opportunity cost is ignored.(c) the opportunity cost is transferred from theconsumers of the product to the tax-payingpublic.(d) the opportunity cost is transferred from theconsumers of the product to the Government.

d Microeconomics

Opportunity cost is defined as the 'valueof the best alternative that is foregonewhen a particular course of action isundertaken'. If a commodity is providedfree to the public by the Government, then the opportunity cost is transferred fromthe consumers of the product to theGovernment. Opportunity cost involves themeasurement of sacrifices.When thegovernment provides commodity free topublic, it sacrifices a part of its resourceswhich could have been used for secondbest alternative.

https://www.investopedia.com/terms/o/opportunitycost.asphttps://www.investopedia.com/terms/s/subsidy.asp

Difficult

58

Increase in absolute and per capita real GNP do notconnote a higher levelof economic development, if(a) industrial output fails to keep pace with agriculturaloutput.(b) agricultural output fails to keep pace with industrialoutput.(c) poverty and unemployment increase.(d) imports grow faster than exports.

c Macroeconomics An increase in poverty and unemployment cannot result in economic development.

https://www.investopedia.com/articles/economics/12/okuns-law.asp Medium

59

Consider the following statements :Human capital formation as a concept is betterexplained in terms of a processwhich enables

1. individuals of a country to accumulate more capital.2. increasing the knowledge, skill levels and capacitiesof the people of the country.3. accumulation of tangible wealth.4. accumulation of intangible wealth.

Which of the statements given above is/are correct?(a) 1 and 2(b) 2 only(c) 2 and 4(d) 1, 3 and 4

c Microeconomics

Physical capital is tangible and can beeasily sold in the market like any othercommodity. Human capital is intangible; itis endogenously built in the body andmind of its owner.The economists sub-divide intangiblecapital into two major categories:‘education’ (consisting of human capital,including raw labor plus the sum ofknowledge, skills and know how of thepopulation); and ‘social capital’(represented by beliefs, levels of trust,attitudes, behaviors and the quality offormal and informal institutional infra-structure—including stability,transparency and other elements). Thus both statement 2 and statement 4explain human capital formation.

http://ncert.nic.in/ncerts/l/keec105.pdfhttp://developeconomies.com/development-economics/the-intangible-wealth-of-nations/http://econlog.econlib.org/archives/2005/12/intangible_weal_1.htmlhttps://difficultrun.nathanielgivens.com/2018/04/14/total-and-intangible-wealth-world-bank-report-2018/https://www.americanambassadors.org/publications/ambassadors-review/fall-2008/quantifying-the-wealth-of-nations-the-impact-of-intangible-capital-and-implications-for-policy-forumulation

Difficult

60

Despite being a high saving economy, capital formationmay not result insignificant increase in output due to

(a) weak administrative machinery(b) illiteracy(c) high population density(d) high capital-output ratio

d Basic Concepts of Economy

Capital output ratio is the amount ofcapital needed to produce one unit ofoutput. Hence, a higher capital –outputratio reflects inefficiency in the economy.It is a quantified term which necessarilyimplies reduction in output. Other optionsmay/ may not lead to reduction in output.

https://www.businesstoday.in/union-budget-2018-19/columns/analyzing-budget-2018-and-the-saving-investment-climate/story/270422.html

Medium

Page 29: CSE-PRELIMS 2018 • PAPER-1 · 2 | Detailed of Solutions of CSE-Prelims:2018 • Paper-1 Analysis of 2018 Prelims: 1. The question paper was difficult as compared to the last year’s

Detailed of Solutions of CSE-Prelims:2018 • Paper-1 | 29

61

After the Santhal Uprising subsided, what was/were themeasure/measures taken by the colonial government

1. The territories called `Santhal Paraganas’ werecreated.2. It became illegal for a Santhal to transfer land to a non-Santhal.

Select the correct answer using the code given below :(a) 1 only(b) 2 only(c) Both 1 and 2(d) Neither 1 nor 2

c Tribal and Peasants Uprising

British introduced a special system ofadministration and for this Act XXXVII of1855 was passed to suppress this tribaldominated political unrest rather thangoing to root to solve the problem.Statement 1 is correct :A separatedistrict of Santhal Parganas was formed.Statement 2 is correct : Land transferwas declared illegal as per tenancy law.

http://yojana.gov.in/tribal-land-rights.asphttp://indianexpress.com/article/india/politics/explained-the-law-political-stakes/

Difficult

62

Economically, one of the results of the British rule inIndia in the 19thcentury was the

(a) increase in the export of Indian handicrafts(b) growth in the number of Indian owned factories(c) commercialization of Indian agriculture(d) rapid increase in the urban population

c Impacts of British Rule

Commercialisation of agriculture startedwhen industrial develpment took pace inEngland. Moreover it became prominentaround 1860s especially in cotton andindigo .

Modern India NCERT by Bipin Chandra Easy

63

If the President of India exercises his power as providedunder Article 356 of the Constitution in respect of aparticular State, then

(a) the Assembly of the State is automatically dissolved.(b) the powers of the Legislature of that State shall beexercisable by or under the authority of the Parliament.(c) Article 19 is suspended in that State.(d) the President can make laws relating to that State.

b President

When the President’s Rule is imposed ina state the President acquires theextraordinary power to declare that thepowers of the state legislature are to beexercised by the Parliament .

Laxmikanth, 5th edition Page 16.6 Easy

Prelims Test Series: Test 26Q.67 Consider the following statements aboutthe ‘Proclamation of Emergency’:1. All types of emergency have been enforced inthe history of the Indian Republic.2. The Proclamation of the President’s Rulesimultaneously results in power of the Legislatureof the State be exercised by or under theauthority of Parliament.Which of the statements given above is/arecorrect?(a) 1 only (b) 2 only(c) Both 1 and 2 (d) Neither 1 nor 2

64

Consider the following pairs : Craft Heritage of1. Puthukkuli shawls Tamil Nadu2. Sujni embroidery Maharashtra3. Uppada Jamdani Saris Karnataka

Which of the pairs given above is/are correct ?(a) 1 only(b) 1 and 2(c) 3 only(d) 2 and 3

a Crafts

Pair one is correctly matched :Puthukkuli shawls are made by Todas ofNilgiri hills in Tamil Nadu. It is locallycalled Pugur, meaning flower, the fineand intricate Toda embroidery is done bytribal men and women on shawls. Pair 2 and 3 are not correct: Uppada Sarees are a distinctive types of sareesand dress materials woven in Uppadatown of Andhra Pradesh.Sujani (or Sujini) is one of the mostpopular form of conventional art and craftprevailing in Bihar.

http://www.lepakshihandicrafts.gov.in/category-uppada.htmlhttp://www.umsas.org.in/en/bihar-arts-crafts/sujini-embroidery/

Difficult

Page 30: CSE-PRELIMS 2018 • PAPER-1 · 2 | Detailed of Solutions of CSE-Prelims:2018 • Paper-1 Analysis of 2018 Prelims: 1. The question paper was difficult as compared to the last year’s

30 | Detailed of Solutions of CSE-Prelims:2018 • Paper-1 65

In which of the following areas can GPS technology beused ?

1. Mobile phone operations2. Banking operations3. Controlling the power grids

Select the correct answer using the code given below :(a) 1 only(b) 2 and 3 only(c) 1 and 3 only(d) 1, 2 and 3

d Techonology

It can be used in all the three mentionedopertations :1.Cell phones with GPS receiverscommunicate with units from among the30 global positioning satellites in the GPSsystem. This has been used for trackingand security measurements.2.In the banking industry, live GPStracking devices are placed on cashbundles or boxes so that money can betraced to and from armoured vehicles andvaults.3.Power system owners and operatorsuse GPS for navigation, position andtiming

https://www.sciencedirect.com/science/article/pii/S1877042813024397https://www.brighthub.com/electronics/gps/articles/93343.aspxhttps://www.gps.gov/cgsic/meetings/2016/silverstein.pdf

Easy

66

Consider the following statements :

1. The Reserve Bank of India manages and servicesGovernment of India Securities but not any StateGovernment Securities.2. Treasury bills are issued by the Government of Indiaand there are no treasury bills issued by the StateGovernments.3. Treasury bills offer are issued at a discount from thepar value.

Which of the statements given above is/are correct?(a) 1 and 2 only(b) 3 only(c) 2 and 3 only(d) 1, 2 and 3

c Banking

statement 2 and 3 are correct butstatement 1 is not correct as explainedbelow.RBI manages G-Secs of both the UnionGovernment and the state governments.In India, the Central Government issuesboth, treasury bills and bonds or datedsecurities while the State Governmentsissue only bonds or dated securities,which are called the State DevelopmentLoans (SDLs).Treasury bills are zero coupon securitiesand pay no interest. They are issued at adiscount and redeemed at the face valueat maturity. For example, a 91 dayTreasury bill of ₹100/- (face value) maybe issued at say ₹ 98.20, that is, at adiscount of say, ₹1.80 and would beredeemed at the face value of ₹100/-.

https://m.rbi.org.in/scripts/FAQView.aspx?Id=79 Difficult

Prelims Test Series : Test 26

Q.81 Consider the following statements withrespect to ‘Treasury Bills’:1. It is issued on behalf of the government by theRBI2. It is a long-term debt obligationWhich of the statements given above is/arecorrect.(a) 1 only (b) 2 only(c) Both 1 and 2 (d) Neither 1 nor 2

Test 28Q.21 Consider the following statements:1. Treasury bills are zero coupon bonds.2. They are offered on a discount rate.Which of the statements given above is/arecorrect?(a) 1 only (b) 2 only(c) Both 1 and 2 (d) Neither 1 nor 2

67

Consider the following statements :

1. The Earth’s magnetic field has reversed every fewhundredthousand years.2. When the Earth was created more than 4000 millionyears ago, there was 54% oxygen and no carbondioxide.3. When living organisms originated, they modified theearly atmosphere of the Earth.

Which of the statements given above is/are correct ?(a) 1 only(b) 2 and 3 only(c) 1 and 3 only(d) 1, 2 and 3

c Physical Geography

Both the statement 1 and 3 are correctbut statement 2 is not correct.The early atmosphere was dominated byhydrogen and helium.There are three stages in the evolution ofthe present atmosphere. The first stage ismarked by the loss of primordialatmosphere.In the second stage, the hotinterior of the earth contributed to theevolution of the atmosphere. Finally, thecomposition of the atmosphere wasmodified by the living world through theprocess of photosynthesis

https://www.nasa.gov/topics/earth/features/2012-poleReversal.html

http://ncert.nic.in/ncerts/l/kegy202.pdf

Medium

68

The terms ‘WannaCry, Petya and EternalBlue’sometimes mentioned inthe news recently are related to(a) Exoplanets(b) Cryptocurrency(c) Cyber attacks(d) Mini satellites

c Techonology

WannaCry and Petya were cyberattacksin form of ransomware that were in thenews last year.

EternalBlue was an exploit in olderWindows systems released by TheShadowBrokers a few months prior to the attack.

http://www.thehindu.com/todays-paper/tp-opinion/held-to-ransomware/article19167509.ece

Easy

Prelims Test Series : Test 6Q.26 The terms ‘Petya’ and ‘Bad Rabbit’ aresometime seen in the news. Which of thefollowing best describe these terms?(a) Critically endangered species(b) Super bugs(c) Interceptor missiles(d) Ransom-ware

Magazine (Current affairs Made Easy)WannaCry is mentioned in January 2018 edition (Page 54)

69

With reference to the circumstances in Indianagriculture, the concept of“Conservation Agriculture” assumes significance. Whichof the following fallunder the Conservation Agriculture ?

1. Avoiding the monoculture practices2. Adopting minimum tillage3. Avoiding the cultivation of plantation crops4. Using crop residues to cover soil surface5. Adopting spatial and temporal crop sequencing/croprotations

Select the correct answer using the code given below :(a) 1, 3 and 4(b) 2, 3, 4 and 5(c) 2, 4 and 5(d) 1, 2, 3 and 5

c Physical Geography

Conservation Agriculture is a farmingsystem that promotes maintenance of apermanent soil cover, minimum soildisturbance (i.e. no tillage), anddiversification of plant species. Avoiding apractice like monoculture, plantation etc isnot a conservation step. Monoculturepractices can be improved upon to reduceits ecological impact. ConservationAgriculture does not include avoiding thepractice itself.Hence statement 1 and statement 3 arenot applicable under conservationpractice.

http://www.fao.org/3/a-i7480e.pdf Difficult

Page 31: CSE-PRELIMS 2018 • PAPER-1 · 2 | Detailed of Solutions of CSE-Prelims:2018 • Paper-1 Analysis of 2018 Prelims: 1. The question paper was difficult as compared to the last year’s

Detailed of Solutions of CSE-Prelims:2018 • Paper-1 | 31

70

The term “sixth mass, extinction/sixth extinction” is oftenmentioned in the news in the context of the discussion of

(a) Widespread monoculture practices in agriculture andlargescale commercial farming withindiscriminate use of chemicals in many parts of theworld that may result in the loss of goodnative ecosystems.(b) Fears of a possible collision of a meteorite with theEarth in the near future in the manner it happened 65million years ago that caused the mass extinction ofmany species including those of dinosaurs.(c) Large scale cultivation of genetically modified cropsin many parts of the world and promoting their cultivationin other parts of the world which may cause thedisappearance ofgood native crop plants and the loss of food biodiversity.(d) Mankind’s over-exploitation/ misuse of naturalresources, fragmentation/loss of naturalhabitats, destruction of ecosystems, pollution and globalclimate change.

d Ecology

The Holocene extinction,otherwisereferred to as the Sixth extinction orAnthropocene extinction, is the ongoingextinction event of species during thepresent Holocene epoch, mainly as aresult of human activity.

http://www.thehindu.com/todays-paper/tp-national/earth-facing-sixth-mass-extinction/article19260792.ece

Medium

71

Consider the following statements :1. Capital Adequacy Ratio (CAR) is the amount thatbanks have to maintain in the form of their own funds tooffset any loss that banks incur if the account-holders failto repay dues.2. CAR is decided by each individual bank

Which of the statements given above is/are correct ?(a) 1 only(b) 2 only(c) Both 1 and 2(d) Neither 1 nor 2

a Eonomy Concepts

Statement 1 is correct : The reasonminimum capital adequacy ratios (CARs)are critical is to make sure that bankshave enough cushion to absorb areasonable amount of losses before theybecome insolvent and consequently losedepositors’ funds. The capital adequacyratios ensure the efficiency and stability ofa nation’s financial system by lowering therisk of banks becoming insolvent.Statement 2 is also not correct: Indianscheduled commercial banks are requiredto maintain a CAR as per the RBI norms.

https://www.investopedia.com/terms/c/capitaladequacyratio.asphttps://economictimes.indiatimes.com/definition/capital-adequacy-ratio

Medium

72

The identity platform Aadhaar’ provides open“Application Programming Interfaces (APIs)”. What doesit imply ?

1. It can be integrated into any electronic device.2. Online authentication using iris is possible.

Which of the statements given above is/are correct ?(a) 1 only(b) 2 only(c) Both 1 and 2(d) Neither 1 nor 2

c Techonology

statement 1 is correct: An applicationprogramming interface (API) is a set ofsubroutine definitions, protocols, andtools for building application software. Ingeneral terms, it is a set of clearly definedmethods of communication betweenvarious software components.The APIcan be integrated into any electronicdevice.statement 2 is correct : Iris scan is oneof the possible modes of Aadharauthentication.

https://aadhaarapi.com/,https://uidai.gov.in/images/FrontPageUpdates/aadhaar_authentication_api_2_0.pdf

Medium

Page 32: CSE-PRELIMS 2018 • PAPER-1 · 2 | Detailed of Solutions of CSE-Prelims:2018 • Paper-1 Analysis of 2018 Prelims: 1. The question paper was difficult as compared to the last year’s

32 | Detailed of Solutions of CSE-Prelims:2018 • Paper-1

73

Very recently, in which of the following countries havelakhs of people either suffered from severe famine/acutemalnutrition or died due to starvation caused bywar/ethnic conflicts ?

(a) Angola and Zambia(b) Morocco and Tunisia(c) Venezuela and Colombia(d) Yemen and South Sudan

d Mapping

South Sudan is in the midst of acatastrophic humanitarian crisis driven bythree years of brutal civil war. Nearly halfof the population are facing extremehunger and are in need of urgent aid.Yemen, one of the Arab world's poorestcountries, has been devastated by a civilwar since 2015.

https://www.oxfam.org/en/emergencies/hunger-crisis-south-sudan

http://www.bbc.com/news/world-middle-east-29319423

Easy

Magazine (Current affairs Made Easy)May 2018 edition (Page 60)

74

Regarding Wood’s Dispatch, which of the followingstatements are true ?

1. Grants-in-Aid system was introduced.2. Establishment of universities was recommended.3. English as a medium of instruction at all levels ofeducation was recommended.

Select the correct answer using the code given below :(a) 1 and 2 only(b) 2 and 3 only(c) 1 and 3 only(d) 1, 2 and 3

a Education

Statement 3 is not correct .As per theWood's Dispatch,the medium ofinstruction at the primary level was to bevernacular while at the higher levels itwould be English. As per this despatch:1.An education department was to be setin every province.2.Universities on the model of the LondonUniversity be established in big citiessuch as Bombay, Calcutta and Madras.3.Affiliated private schools should begiven grant in aid.4.The Indian natives should be giventraining in their mother tongue also

http://www.nios.ac.in/media/documents/dled/Block1_501.pdf

Medium

Prelims Test Series: Test 21Q18- ExplanationThe major landmark in the development ofEnglish education during the British period wasthe Wood’s Despatch of 1854. Sir CharlesWood, the president of the Board of Control, in1854 laid down the policies which became theguiding principles of the education programme ofthe government of India.The major recommendations of the Despatchwere as follows:• The creation of a department of publicinstruction in each of the five provinces of thecompany’s territory,• The establishment of university at Calcutta,Bombay a& Madras,• The establishment of a network of gradedschools-high schools, middle schools and theelementary schools,• The establishment of teachers traininginstitutions,• The promotion of vernacular schools,• The introduction of a system of grants-in-aid forfinancial held to the schools, etc.

75

With reference to the Parliament of India, which of thefollowing Parliamentary Committees scrutinizes andreports to the House whether the powers to makeregulations, rules, sub-rules, by-laws, etc. conferred bythe Constitution or delegated by the Parliament arebeing properly exercised by the Executive within thescope of such delegation ?

(a) Committee on Government Assurances(b) Committee on Subordinate Legislation(c) Rules Committee(d) Business Advisory Committee

b Parliamentary Committees

Amongst the mechanisms evolved by thelegislature to exercise control over thedelegated legislation, the most importantis the constitution of the Committee onSubordinate Legislation. It is thisCommittee of the legislature whichexamines if the powers conferred by theConstitution or delegated under an Actpassed by the legislature have been dulyexercised and are within the confermentor delegation, and not beyond. It has tosee that delegated legislation does nottransgress into areas not prescribed for it,and also that it does not venture tointrude into the sphere which is the soleconcern of the legislature itself.

https://rajyasabha.nic.in/rsnew/practice_procedure/book13.asp

Medium

Page 33: CSE-PRELIMS 2018 • PAPER-1 · 2 | Detailed of Solutions of CSE-Prelims:2018 • Paper-1 Analysis of 2018 Prelims: 1. The question paper was difficult as compared to the last year’s

Detailed of Solutions of CSE-Prelims:2018 • Paper-1 | 33

76

Consider the following statements:

1. As per the Right to Education (RTE) Act, to be eligiblefor appointment as a teacher in a State, a person wouldbe required to possess the minimum qualification laiddown by the concerned State Council of TeacherEducation.2. As per the RTE Act, for teaching primary classes, acandidate is required to pass a Teacher Eligibility Testconducted in accordance with the National Council ofTeacher ducation guidelines.3. In India, more than 90% of teacher educationinstitutions are directly under the State Governments.

Which of the statements given above is/are correct ?(a) 1 and 2(b) 2 only(c) 1 and 3 only(d) 3 only

b Governance

Statement 1 is not correct: The CentralGovernment has notified the NationalCouncil for eacher Education (NCTE) asthe academic authority for prescribingteacher qualifications.tatement 2 is correct : As per Section23(1) of the RTE Act, 2009, the NationalCouncil for eacher Education (NCTE) haslaid down the minimum qualifications for aperson to be eligible for appointment as ateacher in class I to VIII.One of the essential qualifications for aperson to be eligible for appointment as ateacher is that he/she should pass theTeacher Eligibility Test (TET) which willbe conducted by the appropriateGovernment.Statement 3 is not correct: More than90% of teacher education institutions arenot directly under the State Governments.

http://mhrd.gov.in/sites/upload_files/mhrd/files/upload_document/RTE_Section_wise_rationale_rev_0.pdfhttp://mhrd.gov.in/sites/upload_files/mhrd/files/upload_document/RTE_TET.pdf

Difficult

Page 34: CSE-PRELIMS 2018 • PAPER-1 · 2 | Detailed of Solutions of CSE-Prelims:2018 • Paper-1 Analysis of 2018 Prelims: 1. The question paper was difficult as compared to the last year’s

34 | Detailed of Solutions of CSE-Prelims:2018 • Paper-1

77

Consider the following pairs: Traditions : State 1. Chapchar Kut festival : Mizoram 2. Khongjom Parba ballad : Manipur 3. Thang-Ta dance : Sikkim

Which of the pairs given above is/are correct ? (a) 1 only (b) 1 and 2 (c) 3 only (d) 2 and 3

b Traditions

Chapchar Kut festival: It is a festival ofMizoram.The festival is held during theperiod when the bamboos and trees thathave been cut down are being awaited todry to be burnt for jhumming. During thisbrief layoff period of jhumming, the Mizoancestors could have all the time forthemselves. They spend their leisurehunting games, fishing, et al. Khongjom Parba ballad: It is a style ofballad singing from Manipur that depictsstories of the heroic battle fought byManipuris against British forces in 1891.Thang-ta dance: It is a martial dance ofManipur not Sikkim.

http://mizoram.nic.in/about/chapcharkut.htm

http://manipurtimes.com/reviving-the-dying-traditional-manipuri-music-khongjom-parva-padma-shri-awardee-nameirakpam-ibemni/

http://ccrtindia.gov.in/manipuri.php

Difficult

Prelims Test Series-Test Number 18-QuestionNumber 78Q.78 Consider the following pairs: Festival State1. Kambala : Karnataka2. Chapchar Kut : Mizoram3. Pooram : Andhra Pradesh4. Karam Puja : JharkhandWhich of the pairs given above is/are correctlymatched?(a) 1 and 2 only (b) 1, 2 and 4 only(c) 3 and 4 only (d) 1, 2, 3 and 4There are few days of relaxation before the seriousbusiness of sowing starts and that is when the ChapcharKut festival is celebrated in Mizoram with gaiety andfervour. A spring festival, this is the most importantfestival and the only one regularly observed during thefirst week of March in Mizoram.Prelims Test Series-Test Number 26-QuestionNumber 79_ExplanationPair 4 is correct: As per Mizo custom during theChapchar Kut festival not only singing, dance shouldalso continue throughout the festival. Special occasionfor singing and dancing is called ‘chai’ and songs areknown as ‘chai hia’ (chai songs).

Prelims Test Series-Test Number 3-QuestionNumber 49Under the revamped "Khelo India" proposal, there is anexclusive component called "Promotion of Rural,Indigenous and Tribal Games" for promotion anddevelopment of traditional sports. In order to revive suchtraditional sports and games, to promote them amongthe masses, SportsAuthority of India (SAI) has adopted the followingindigenous games disciplines for their promotion:• Kalaripayatu, Kerala • Silambam, Tamilnadu• abaddi, Telengana • Archery, Jharkhand• Malkhamb, Maharashtra • Mukna, Imphal• Thangta, Imphal • Khomlainai, Assam• Gatka, Punjab

Page 35: CSE-PRELIMS 2018 • PAPER-1 · 2 | Detailed of Solutions of CSE-Prelims:2018 • Paper-1 Analysis of 2018 Prelims: 1. The question paper was difficult as compared to the last year’s

Detailed of Solutions of CSE-Prelims:2018 • Paper-1 | 35

78

Consider the following statements :

1. The Food Safety and Standards Act, 2006 replacedthe Prevention of Food Adulteration Act, 1954.2. The Food Safety and Standards Authority of India(FSSAI) is under the charge of Director General ofHealth Services in the Union Ministry of Health andFamily Welfare.

Which of the statements given above is/are correct ?(a) 1 only(b) 2 only(c) Both 1 and 2(d) Neither 1 nor 2

a Governance

Statement 1 is correct: The Food Safetyand Standards Authority of India (FSSAI)has been established under Food Safetyand Standards , 2006 which consolidatesvarious acts & orders that have hithertohandled food related issues in variousMinistries and Departments.The Act consolidates the followingacts and orders:• Prevention of Food AdulterationAct,1954,• Fruit Products Order , 1955, • Meat Food Products Order,1973,• Vegetable Oil Products (Control) Order,1947,• Edible Oils Packaging(Regulation)Order 1988, • Solvent Extracted Oil, De- Oiled Mealand Edible Flour (Control) Order, 1967, -Milk and Milk Products Order, 1992 Statement 2 is not correct: FSSAI is anautonomous body. Ministry of Health &Family Welfare, Government of India isthe Administrative Ministry for theimplementation of FSSAI. It is not underthe charge of Director General of HealthServices.

http://www.fssai.gov.in/home/about-us/introduction.htmlhttp://www.dnaindia.com/business/report-fssai-favours-comprehensive-legislation-for-organic-foods-2597234https://www.fssai.gov.in/dam/jcr:7936048f-5532-403a-add4-10d4e72dd2f5/Circular_FSSAI_Filling_Post_31_08_2017.pdf

Difficult

Prelims Test Series-Test Number 4-QuestionNumber 93Q.93 Food Safety and Standards Authority ofIndia (FSSAI) has recently launched its onlineplatform to make food safety officersaccountable. Consider the following statementsregarding ‘FSSAI’:1. Ministry of Consumer Affairs, Food and PublicDistribution is the Administrative Ministry for theimplementation of FSSAI.2. It provides scientific advice and technicalsupport to Central and State Governments in thematters of framing the policy relatedto food safety.Which of the statements given above is/arecorrect?(a) 1 only (b) 2 only(c) Both 1 and 2 (d) Neither 1 nor 2

93. (b)To bring in transparency in food safety inspectionand sampling, food regulator FSSAI put in placea nationwide online platform and asked states toadopt this system as it would help eliminate\discrepancy and make food safety officersaccountable.Statement 1 is not correct: Ministry of Health &Family Welfare, Government of India is theAdministrative Ministry for the implementation ofFSSAI.Statement 2 is correct: FSSAI has beenmandated by the FSS Act, 2006 for performingvarious functions which also includes providingscientific advice and technical support to CentralGovernment and State Governments in thematters of framing the policy and rules in areaswhich have a direct or indirect bearing of foodsafety and nutrition.

79

The term “two-state solution” is sometimes mentioned inthe news in the context of the affairs of

(a) China(b) Israel(c) Iraq(d) Yemen

b International Relations

The two-state solution refers to a solutionof the Israeli–Palestinian conflict whichcalls for "two states for two groups ofpeoplThe two-state solution refers to asolution of the Israeli–Palestinian conflictwhich calls for "two states for two groupsof people."

https://www.nytimes.com/2016/12/29/world/middleeast/israel-palestinians-two-state-solution.html

Easy

Magazine (Current affairs Made Easy)February 2018 edition (Page 30)

Page 36: CSE-PRELIMS 2018 • PAPER-1 · 2 | Detailed of Solutions of CSE-Prelims:2018 • Paper-1 Analysis of 2018 Prelims: 1. The question paper was difficult as compared to the last year’s

36 | Detailed of Solutions of CSE-Prelims:2018 • Paper-1

80

With reference to the provisions made under theNational Food Security Act, 2013; consider the followingstatements :

1. The families coming under the category of ‘belowpoverty line (BPL)’ only are eligible to receive subsidisedfood grains.2. The eldest woman in a household, of age 18 years orabove, shall be the head of the household for thepurpose of issuance of a ration card.3. Pregnant women and lactating mothers are entitled toa ‘takehome ration’ of 1600 calories per day duringpregnancy and for six months thereafter.

Which of the statements given above is/are correct ?(a) 1 and 2(b) 2 only(c) 1 and 3(d) 3 only

b Governance

Statetement 1 is not correct: The Actdefines ‘eligible households’ under twocategories: (i) households covered underthe Antyodaya Anna Yojana (AAY); and(ii) households covered as the priorityhouseholds under the Targeted PublicDistributionSystem (TPDS). Statetement 2 is correct: The eldestwoman of eighteen years of age or abovein every eligible household, whereveravailable, shall be head of the householdfor the purpose of issuance of ration card. Statetement 3 is not correct: It is 600KCal not 1600 Calories.

https://rajyasabha.nic.in/rsnew/publication_electronic/National_Food_security_Act2013.pdf

Medium

Prelims Test Series-Test Number 5-Question Number 100Q.100 National Food Security Act allocates subsidized foodgrains to:(a) All Indian citizens.(b) Only Below Poverty Line citizens.(c) All agricultural households.(d) A certain fixed percentage of urban and rural population.100. (d)Ministry of Consumer Affairs, Food and Public Distributionachieved new milestone during 2013 with the enactment ofhistoric NationalFood Security Act. The largest social welfare programme ofthe world would provide legal right to about 82 crore peoplefor subsidized foodgrains. As passed by the Parliament, theGovernment notified the National Food Security Act, 2013 on10th September, 2013 with the objective to provide for foodand nutritional security in human life cycle approach, byensuring access to adequate quantity of quality food ataffordable prices to people to live a life with dignity. The Actprovides for coverage of upto 75% of the rural opulation andupto 50% of the urban population for receiving subsidizedfoodgrains under Targeted Public Distribution System(TPDS), thus covering about twothirds of the population.Persons belonging to eligible households will be entitled toreceive 5 Kilograms of foodgrains per person per monthat subsidised prices of Rs. 3/2/1 per Kg for rice/wheat/coarse grains, provided that existing Antyodaya AnnaYojana (AAY) households,which constitute the poorest of the poor, will continue toreceive 35 Kgs of foodgrains per household per month.Eligible households means households covered under thepriority households and the Antyodaya Anna Yojana.The Act also has a special focus on the nutritional support towomen and children. Besides meal to pregnant women andlactating mothers duringpregnancy and six months after the child birth, such womenwill also be entitled to receive maternity benefit of not lessthan Rs. 6,000.Children upto 14 years of age will be entitled to nutritiousmeals or take home rations as per the prescribed nutritionalstandards.

Page 37: CSE-PRELIMS 2018 • PAPER-1 · 2 | Detailed of Solutions of CSE-Prelims:2018 • Paper-1 Analysis of 2018 Prelims: 1. The question paper was difficult as compared to the last year’s

Detailed of Solutions of CSE-Prelims:2018 • Paper-1 | 37

81

India enacted The Geographical Indications of Goods(Registration and Protection) Act, 1999 in order tocomply with the obligations to (a) ILO(b) IMF(c) UNCTAD(d) WTO

d Intellectual Property

India, as a member of the World TradeOrganization (WTO), enacted theGeographical Indications of Goods(Registration & Protection)Act, 1999 hascome into force with effect from 15thSeptember 2003.

http://www.ipindia.nic.in/about-us-gi.htm Easy

Prelims Test Series-Test Number 11-QuestionNumber 76Q.76 ‘Banaganapalle Mango’ of Andhra Pradesh wasrecently accorded ‘Geographical Indications’ (GI) tag.With reference to GI, which of the following statementsis/are correct?1. In India, Geographical Indications registration isadministered by theGeographical Indications of goods (Registration andProtection) Act, 1999. 2. At the International level, GI is governed by WorldTrade Organisation’s (WTO’s) Agreement on Trade-Related Aspects of Intellectual Property Rights (TRIPS).Which of the statements given above is/are correct?(a) 1 only (b) 2 only(c) Both 1 and 2 (d) Neither 1 nor 2

76. (c)A GI tag certifies the origin of a product or produce froma particular region as the quality or other features of theproduct is attributable only to the place of its origin. Thetag helps farmers or manufacturers, as the case may be,to get a better price in the market. Statement 1 iscorrect: In India, Geographical Indications registration isadministered by the Geographical Indications of goods(Registration and Protection) Act, 1999 which came intoforce with effect from September 2003. This act isadministered by the controller general of patents,designs and trademarks, who is also the registrar ofgeographical indications. Statement 2 is correct: GI isgoverned by World Trade Organisation’s(WTO’s)Agreement on Trade-Related Aspects ofIntellectual Property Rights (TRIPS).

Page 38: CSE-PRELIMS 2018 • PAPER-1 · 2 | Detailed of Solutions of CSE-Prelims:2018 • Paper-1 Analysis of 2018 Prelims: 1. The question paper was difficult as compared to the last year’s

38 | Detailed of Solutions of CSE-Prelims:2018 • Paper-1

82

Consider the following statements :

1. In India, State Governments do not have the power toauction non-coal mines.2. Andhra Pradesh and Jharkhand do not have goldmines.3. Rajasthan has iron ore mines.

Which of the statements given above is/are correct ?(a) 1 and 2(b) 2 only(c) 1 and 3(d) 3 only

d Distribution of Resources

Statement 1 is not correct: Stategovernments have the power to auctionnon-coal mines. For example- Limestoneauctions by Chhattisgarh,AndhraPradesh,Gujarat and others.Statement 2 is not correct: Jharkhand has gold mines like Rungta mines whoseauction was held last year. Statement 3 is correct: Rajasthan hasiron ore mines in Bhilwara.

https://www.financialexpress.com/industry/95-non-coal-mines-may-be-auctioned-in-fy19/1115556/

https://timesofindia.indiatimes.com/city/delhi/Jharkhand-ready-to-auction-gold-mines-next-month/articleshow/51925705.cms

https://community.data.gov.in/working-and-non-working-iron-ore-mines-in-various-states-as-on-20-04-2016/

http://indianexpress.com/article/business/business-others/non-coal-blocks-states-undertake-groundwork-for-auction-of-71-mines-this-year-4525263/

Difficult

83

With reference to digital payments, consider thefollowing statements :

1. BHIM app allows the user to transfer money to anyonewith a UPI-enabled bank account.2. While a chip-pin debit card has four factors ofauthentication, BHIM app has only two factors ofauthentication.

Which of the statements given above is/are correct ?

(a) 1 only(b) 2 only(c) Both 1 and 2(d) Neither 1 nor 2

a Technology/Digital India

Statement 1 is correct: Bharat Interfacefor Money (BHIM) is a payment app thatlets you make simple, easy and quicktransactions using Unified PaymentsInterface (UPI). You can make direct bankpayments to anyone on UPI using theirUPI ID or scanning their QR with theBHIM app. You can also request moneythrough the app from a UPI ID.Statement 2 is not correct: Three-factorauthentication is key safety feature on theBHIM app.

https://www.bhimupi.org.in/who-we-are

http://indianexpress.com/article/technology/tech-news-technology/bhim-app-narendra-modi-upi-online-transaction-safety-4453599/

Difficult

84

Among the following cities, which one lies on a longitudeclosest to that of Delhi?

(a) Bengaluru(b) Hyderabad(c) Nagpur(d) Pune

a Mapping

The longitudes of the cities are: Delhilongitude:77.1025° E, Bangalore:77.5946° EHyderabad's longitude is 78.4867° ENagpur's longitude is 79.0882° EPune's longitude is 73.8567° E

Atlas Difficult

85

International Labour Organisation’s Conventions 138and 182 are related to(a) Child labour(b) Adaptation of agricultural practices to global climatechange(c) Regulation of food prices and food security(d) Gender parity at the workplace

a International Organization

The two conventions are:ILO Convention No. 182 on the worstforms of child labourILO Convention No. 138 on the minimumage for admission to employment andwork.

http://www.ilo.org/ipec/facts/ILOconventionsonchildlabour/lang--en/index.htm

Easy

Magazine (Current affairs Made Easy)November, 2017 edition (Page 16)

Page 39: CSE-PRELIMS 2018 • PAPER-1 · 2 | Detailed of Solutions of CSE-Prelims:2018 • Paper-1 Analysis of 2018 Prelims: 1. The question paper was difficult as compared to the last year’s

Detailed of Solutions of CSE-Prelims:2018 • Paper-1 | 39

86

Consider the following statements

1. In the first Lok Sabha, the single largest party in theopposition was the Swatantra Party.2. In the Lok Sabha, a “Leader of the Opposition” wasrecognised for the first time in 1969.3. In the Lok Sabha, if a party does not have a minimumof 75 members, its leader cannot berecognised as the Leader of the Opposition.

Which of the statements given above is/are correct ?(a) 1 and 3 only(b) 2 only(c) 2 and 3 only(d) 1, 2 and 3

b Lok Sabha

Statement 1 is not Correct: In the firstLok Sabha, the single largest party in theopposition was Communist Party of India.Swatantra Party was founded in 1959 byC. Rajagopalachari.He parted ways withthe Congress in 1957 after beingdisillusioned by the path it was taking. Hefounded the Swatantra Party in 1959,which favoured classical liberal principlesand free enterprise.Thus the party did notparticipate in the first Lok Sabhaelections.Statement 2 is correct: In the LokSabha, a “Leader of the Opposition” wasrecognised forthe first time in 1969. Leader of Oppsitionis not a constitutional authority.Statement 3 is not correct: The optionscan be easily eliminated by statement 3,which is incorrect. A minimum of 10%members of the total membership of thehouse is required by a party for its leaderto be recognised as Leader ofOpposition.

Indian Polity by Laxmikant

https://www.livemint.com/Sundayapp/XlvTGlfJcdJu9mQGZcksTI/C-Rajagopalachari--Why-Swatantra.html

Easy

87

Which of the following leaf modifications occur(s) in thedesert areas to inhibit water loss ?

1. Hard and waxy leaves2. Tiny leaves3. Thorns instead of leaves

Select the correct answer using the code given below.(a) 2 and 3 only(b) 2 only(c) 3 only(d) 1, 2 and 3

d Ecology

Desert plants lose very little water throughtranspiration. The leaves in desert plantsare either absent, very small, or they arepresent in the shape of spines. This helpsin reducing loss ofwater from the leaves throughtranspiration.The stem is also covered with a thickwaxy layer, which helps to retain water.

http://ncert.nic.in/ncerts/l/fesc109.pdf Medium

Prelims Test Series-Test Number 5-QuestionNumber 25Statement 2 explanation:-Statement 2 is correct: Desert plants have adaptedto the extremes of heat and aridity by using bothphysical and behavioural mechanisms. They havetiny leaves that close their stomata (pores) duringthe day to avoid water loss and open them at nightto absorb moisture. This keeps them from dryingout, but they need CO2 to photosynthesize in daytime. The CO2 taken in at night is stored as an acid.During the day when stomata are closed theyconvert the acid back to CO2 and photosynthesize.

Page 40: CSE-PRELIMS 2018 • PAPER-1 · 2 | Detailed of Solutions of CSE-Prelims:2018 • Paper-1 Analysis of 2018 Prelims: 1. The question paper was difficult as compared to the last year’s

40 | Detailed of Solutions of CSE-Prelims:2018 • Paper-1

88

As per the NSSO 70th Round “Situation Assessment -Survey of Agricultural Households”,

consider the following statements :1. Rajasthan has the highest percentage share ofagricultural households among its ruralhouseholds.2. Out of the total agricultural households in the country,a little over 60 percent; belong to OBCs.3. In Kerala, a little over 60 percent of agriculturalhouseholds reported to have received maximum incomefrom sources other than agricultural activities.

Which of the statements given above is/are correct ?(a) 2 and 3 only(b) 2 only(c) 1 and 3 only(d) 1, 2 and 3

c Agriculture

Statement 1 is correct: Among themajor States, Rajasthan had highestpercentage of agricultural households(78.4 percent) among its rural householdsfollowed by Uttar Pradesh (74.8 percent)and Madhya Pradesh (70.8 percent).Statement 2 is not correct: It is littleover 45% not 60%.Statement 3 is Correct: Agriculturalactivity (cultivation, livestock and otheragricultural activities) was reported to bethe principal source of income for majorityof the households in all the major States,except Kerala where about 61 percent ofthe agricultural households reported tohave earned maximum income fromsources other than agricultural activities

http://mospi.nic.in/sites/default/files/publication_reports/KI_70_33_19dec14.pdf Difficult

Page 41: CSE-PRELIMS 2018 • PAPER-1 · 2 | Detailed of Solutions of CSE-Prelims:2018 • Paper-1 Analysis of 2018 Prelims: 1. The question paper was difficult as compared to the last year’s

Detailed of Solutions of CSE-Prelims:2018 • Paper-1 | 41

89

How is the National Green Tribunal (NGT) different fromthe Central Pollution Control Board (CPCB) ?

1. The NGT has been established by an Act whereas theCPCB has been created by an executive order of theGovernment.2. The NGT provides environmental justice and helpsreduce the burden of litigation in thehigher courts whereas the CPCB promotes cleanlinessof streams and wells, and aims to improve the quality ofair in the country.

Which of the statements given above is/are correct ?(a) 1 only(b) 2 only(c) Both 1 and 2(d) Neither 1 nor 2

b Governance/Bodies

Statement 1 is not correct: The CentralPollution Control Board (CPCB), statutoryorganisation, was constituted inSeptember, 1974 under the Water(Prevention and Control of Pollution) Act,1974.Statement 2 is correct: The NationalGreen Tribunal has been established on18.10.2010 under the National GreenTribunal Act 2010 for effective andexpeditious disposal of cases relating toenvironmental protection andconservation of forests and other naturalresources.Principal Functions of the CPCB, as speltout in the Water (Prevention and Controlof Pollution) Act, 1974, and the Air(Prevention and Control of Pollution) Act,1981, (i) to promote cleanliness of streams andwells in different areas of the States byprevention, control and abatement ofwater pollution, and (ii) to improve the quality of air and toprevent, control or abate air pollution inthe country.

http://cpcb.nic.in/Introduction/ Medium

Test 16-Question Number 58Q.58 Consider the following statements with referenceto ‘Central Pollution Control Board’1. The CPCB, statutory organisation, was constitutedunder the Air (Prevention and Control of Pollution) Act,1981.2. ‘Sameer’, a mobile App developed by CPCB, provideshourly updates on the National Air Quality Index (AQI). Which of the statements given above is/are correct?(a) 1 only (b) 2 only(c) Both 1 and 2 (d) Neither 1 nor 258. (b)Statement 1 is not correct: The Central PollutionControl Board (CPCB), statutory organisation, wasconstituted in September, 1974 under the Water(Prevention and Control of Pollution) Act, 1974. Further,CPCB was entrusted with the powers and functionsunder the Air (Preventionand Control of Pollution) Act, 1981. Statement 2 iscorrect: Sameer,an APP provides hourly updates on theNational Air Quality Index (AQI) published by CPCB. AirQuality Index is a tool for effective communication of airqualityTest-7-Question Number 44. (b)The National Green Tribunal has been established in2010 under the National Green Tribunal Act 2010 for effective and expeditious disposal of cases relating toenvironmental protection and conservation of forestsand other natural resources including enforcement ofany legal right relating to environment and giving reliefand compensation for damages to persons andproperty. Statement 1 is not correct: The Tribunal shallnot be bound by the procedure laid down under theCode of Civil Procedure, 1908, but shall be guided byprinciples of natural justice. Statement 2 is correct: The tribunal is guided by theprinciples of natural justice. The principles of naturaljustice concern procedural fairness and ensure a fairdecision is reached by an objective decision maker.Maintaining procedural fairness protects the rights ofindividuals

Page 42: CSE-PRELIMS 2018 • PAPER-1 · 2 | Detailed of Solutions of CSE-Prelims:2018 • Paper-1 Analysis of 2018 Prelims: 1. The question paper was difficult as compared to the last year’s

42 | Detailed of Solutions of CSE-Prelims:2018 • Paper-1

90

Consider the following statements

1. The Parliament of India can place a particular law inthe Ninth Schedule of the Constitution of India.2. The validity of a law placed in the Ninth Schedulecannot be examined by any court and nojudgement can be made on it

Which of the statements given above is/are correct ?(a) 1 only(b) 2 only(c) Both 1 and 2(d) Neither 1 nor 2

a Polity

Statement 1 is correct: The Parliamentof India can place a particular law in theninth schedule of the Constitution. Once alaw is enacted and included in the NinthSchedule, it gets protection under Article31-B (validation of certain Acts andRegulations).Statement 2 is not correct: In theCoelho Case Supreme Court Bench heldthat all such laws included in the NinthSchedule after April 24, 1973 would betested individually on the touchstone ofviolation of fundamental rights or thebasic structure doctrine. The laws wouldbe examined separately by a three-judgeBench and if these were found to violatethe fundamental rights, abridge orabrogate any of the rights or protectiongranted to the people would be set aside.

http://www.thehindu.com/todays-paper/IX-Schedule-laws-open-to-review/article14705323.ece

Medium

91

Which one of the following best describes the term“Merchant Discount Rate” sometimes seen in news ?

(a) The incentive given by a bank to a merchant foraccepting payments through debit cards pertaining tothat bank. (b) The amount paid back by banks to their customerswhen they use debit cards for financial transactions forpurchasing goods or services. (c) The charge to a merchant by a bank for acceptingpayments from his customers through the bank’s debitcards. (d) The incentive given by the Government to merchantsfor promoting digital payments by their customersthrough Point of Sale (PoS) machines and debit cards.

c Banking

MDR is a fee charged from a merchant bya bank for accepting payments fromcustomers through credit and debit cardsin their establishments. MDR compensates the card issuing bank,the lender which puts the PoS terminaland payment gateways such asMastercard or Visa for their services.

https://economictimes.indiatimes.com/wealth/spend/what-is-merchant-discount-rate/articleshow/62390733.cms

Easy

Prelims Test Series-Test Number 9-QuestionNumber 78

Q.78 With respect to ‘Merchant Discount Rate’(MDR), which one of the following statement iscorrect?(a) It is the rate charged to a merchant by a bankfor providing debit and credit card services.(b) It is the rate paid to the merchant by a bankfor availing debit and credit card services.(c) It is the rate charged to a bank by theReserve Bank for allowing debit and credit cardservices.(d) None of the above.

78. (a)The Reserve Bank of India (RBI) has recentlyreduced the Merchant Discount Rate (MDR) fordebit card transactions and prescribed separatecap for small and large merchants based on theirannual turnover. The move is aimed toencourage merchants to accept online mode ofpayments. MDR is the fee that a merchant has topay to a bank for every transaction which is splitbetween the bank which has issued the card, thelender whose point-of-sale terminal is being usedand payment gateways like MasterCard or Visa.

Magazine (Current affairs Made Easy)January 2018 edition (Page 68)

Page 43: CSE-PRELIMS 2018 • PAPER-1 · 2 | Detailed of Solutions of CSE-Prelims:2018 • Paper-1 Analysis of 2018 Prelims: 1. The question paper was difficult as compared to the last year’s

Detailed of Solutions of CSE-Prelims:2018 • Paper-1 | 43

92

What is/are the consequence/ consequences of acountry becoming the member of the ‘Nuclear SuppliersGroup’ ?

1. It will have access to the latest and most efficientnuclear technologies.2. It automatically becomes a member of “The Treaty onthe Non-Proliferation of NuclearWeapons (NPT)”.

Which of the statements given above is/are correct ?(a) 1 only(b) 2 only(c) Both 1 and 2(d) Neither 1 nor 2

aInternational

Institutions/Partnerships

Statement 1 is correct: The NSG is thetop club of countries which controlsaccess to technology and guards againstproliferation. Its membership is importantfor India to access cutting-edge hightechnology.Statement 2 is not correct: Treaty onthe Non Proliferation of NuclearWeapons(NPT) is one of the conditionsheld to join the NSG. India will notbecome a member of NPT by joiningNSG.

http://indianexpress.com/article/india/indias-membership-to-nsg-delhi-works-on-beijing-to-drop-objections-on-nuclear-suppliers-group-club-entry-5132252/

Easy

Prelims Test Series-Test Number 28 -QuestionNumber- 25Q.78 Which of the following is/are objectives of NuclearNon-Proliferation Treaty?1. Prevent the spread of nuclear weapons and itstechnology.2. Promote cooperation in the peaceful uses of nuclearenergy.3. Nuclear disarmament.Select the correct answer using the code given below:(a) 1 only (b) 1 and 2 only(c) 2 and 3 only (d) 1, 2 and 378. (d)All the given statements are correct: The NPT is alandmark nternational treaty whose objective is toprevent the spread of nuclear weapons and weaponstechnology, to promote cooperation in the peaceful usesof nuclear energy and to further the goal of achievingnuclear disarmament and general and completedisarmament. Test Number 28 -Question Number- 25Recently, the first initiative under the Indo-Russian dealwas taken to develop atomic energy projects in thirdcountries. In this light, India and Russia shall jointlyimplement this Nuclear Power Project in which of thefollowing countries?(a) Afghanistan (b) Bangladesh(c) Nepal (d) Myanmar25. (b)A MoU was signed at Moscow, Russia betweenthe State Atomic Energy Corporation 'Rosatom';the Ministry of Science and Technology ofBangladesh and the Department of AtomicEnergy, India in February-March 2018 ontrilateral cooperation in implementation of theRooppur Nuclear Power Project in Bangladesh.The Rooppur project is the first initiative underan Indo-Russian deal to undertake atomic energyprojects in third countries. India is nota member of the Nuclear Suppliers Group(NSG) and hence cannot participate directly inconstruction of atomic power reactors. Nuclear Power Corporation of India Limited (NPCIL)will play a key role in building a nuclear powerplant on foreign soil with the proposed supplyof equipment and material for the power stationbeing built by Russia in Bangladesh.

Page 44: CSE-PRELIMS 2018 • PAPER-1 · 2 | Detailed of Solutions of CSE-Prelims:2018 • Paper-1 Analysis of 2018 Prelims: 1. The question paper was difficult as compared to the last year’s

44 | Detailed of Solutions of CSE-Prelims:2018 • Paper-1

93

With reference to India’s decision to levy an equalizationtax of 6% on online advertisement services offered bynon-resident entities, which of the following statementsis/are correct ?

1. It is introduced as a part of the Income Tax Act.2. Non-resident entities that offer advertisement servicesin India can claim a tax credit in their home countryunder the “Double Taxation Avoidance Agreements”.

Select the correct answer using the code given below :(a) 1 only(b) 2 only(c) Both 1 and 2(d) Neither 1 nor 2

d Fiscal Policy

Statement 1 is not correct : The levywas introduced in the Budget as part ofthe finance bill and not as a part ofIncome Tax Act. Statement 2 is not correct: Thecompanies bearing the brunt of the levywould not be able to enjoy the benefit oftax treaties to avoid double taxation intheir home countries.

https://www.business-standard.com/article/economy-policy/finance-ministry-to-clarify-on-equalisation-levy-116030700003_1.html

Difficult

94

Consider the following statements:

1. The Fiscal Responsibility and Budget Management(FRBM) Review Committee Report hasrecommended a debt to GDP ratio of 60% for thegeneral (combined) government by 2023,comprising 40% for the Central Government and 20%for the State Governments.2. The Central Government has domestic liabilities of21% of GDP as compared to that of 49% of GDP of theState Governments.3. As per the Constitution of India, it is mandatory for aState to take the Central overnment’s consent forraising any loan if the former owes any outstandingliabilities to the latter.

Which of the statements given above is/are correct?(a) 1 only(b) 2 and 3 only(c) 1 and 3 only(d) 1, 2 and 3

c Fiscal Policy

Statement 1 is correct: The FiscalResponsibility and Budget Management(FRBM) Review Committee report, haspreferred a debt to GDP ratio of 60% forthe general government by 2023,comprising 40% for the Centralgovernment and 20% for the Stategovernments.Statement 2 is not correct: The Uniongovernment, which has larger domesticliabilities of 49.23% of GDP as comparedto that of the States (21% of GDP).

Statement 3 is correct: Under Clause(3) of Article 293 of Constitution of India, itis mandatory for a State to take theCentral government’s consent for raisingany loan if the former owes anyoutstanding liabilities to the latter.

https://dea.gov.in/sites/default/files/Volume%201%20FRBM%20Review%20Committee%20Report.pdf

Medium

Page 45: CSE-PRELIMS 2018 • PAPER-1 · 2 | Detailed of Solutions of CSE-Prelims:2018 • Paper-1 Analysis of 2018 Prelims: 1. The question paper was difficult as compared to the last year’s

Detailed of Solutions of CSE-Prelims:2018 • Paper-1 | 45

95

Consider the following statements:1. The quantity of imported edible oils is more than thedomestic production of edible oils in the last five years.2. The Government does not impose any customs dutyon all the imported edible oils as a special case.

Which of the statements given above is/are correct?(a) 1 only(b) 2 only(c) Both 1 and 2(d) Neither 1 nor 2

a Agriculture

Statement 1 is correct: In FY2016India’s total edible oil demand stood at 24mn tonnes out of which 9 mn tonnes wasmet from domestic production and 15 mntonnes met from imports. The lattervalued at around Rs. 65,000 crore,constituted around 2.5% of India’s totalimport bill. Statement 2 is not correct: Governmentcan impose any customs or import dutyon imported edible seeds to support itsown farmers.

https://economictimes.indiatimes.com/news/economy/agriculture/india-still-highly-dependent-on-edible-oil-imports-icra/articleshow/60360710.cms

https://www.reuters.com/article/india-vegoils/india-raises-import-tax-on-edible-oils-to-highest-in-a-decade-idINKBN1DI05O

Difficult

Prelims Test Series-Test Number 27-QuestionNumber 39

Q.39 Recently, India raised import tax on palm oilto highest in a decade. In which of the following, ‘Palm oil’ finds its use?1. In Bio fuel and Bio lubricants2. In Cosmetic product 3. In Detergents includingsoaps & soap Blends4. In Pharmaceuticals products5. In Chocolate/Ice-creamSelect the correct answer using the code givenbelow:(a) 2 and 3 only (b) 1, 2 and 4 only(c) 4 and 5 only (d) 1, 2, 3, 4 and 5

96

He wrote biographies of Mazzini, Garibaldi, Shivaji andShrikrishna; stayed in America for some time; and wasalso elected to the Central Assembly. He was

(a) Aurobindo Ghosh(b) Bipin Chandra Pal(c) Lala Lajpat Rai(d) Motilal Nehru

c Personalities of Freedom Struggle

Lala Lajpat Rai : Gifted with a perceptivemind, he was a prolific writer andauthored several works like – “UnhappyIndia”, “Young India: An Interpretation”,“History of Arya Samaj”, “England’s Debtto India” and a series of popularbiographies on Mazzini, Garibaldi andSwami Dayanand.

http://pib.nic.in/newsite/mbErel.aspx?relid=148720 Difficult

Page 46: CSE-PRELIMS 2018 • PAPER-1 · 2 | Detailed of Solutions of CSE-Prelims:2018 • Paper-1 Analysis of 2018 Prelims: 1. The question paper was difficult as compared to the last year’s

46 | Detailed of Solutions of CSE-Prelims:2018 • Paper-1

97

Consider the following statements:1. Aadhaar card can be used as a proof of citizenship ordomicile.2. Once issued, Aadhaar number cannot be deactivatedor omitted by the Issuing Authority.

Which of the statements given above is/are correct?(a) 1 only(b) 2 only(c) Both 1 and 2(d) Neither 1 nor 2

d Inclusive Development

Statement 1 is not correct: Aadhaar card isnot a proof of Indian citizenship, the CalcuttaHigh Court has ruled, while rejecting a claimof citizenship on the ground that the accusedpossessed Aadhaar card.Statement 2 is not correct: UIDAI’s updatepolicy version 2.3 (available on the officialwebsite uidai.gov.in) has provisions fordeactivation of Aadhaar, which are givenbelow:a. If within 2 years of attaining age 5, thechild's biometrics are not updated inAadhaar database, his/her Aadhaar numberwill be deactivated (no authenticationpermissible). It will be reactivated oncebiometrics are updated in database.b. If within 2 years of attaining age 15, thechild's biometrics are not updated inAadhaar database, his/her Aadhaar numberwill be deactivated (no authenticationpermissible). It will be reactivated oncebiometrics are updated in database.c. If Resident has not biometricallyauthenticated in 5 years, his/her One TimePassword (OTP) based authenticationservices will be deactivated. They will be re-activated once the resident biometricallyauthenticates.

https://uidai.gov.in/component/fsf/?view=faq&catid=-2&start=160

Medium

Page 47: CSE-PRELIMS 2018 • PAPER-1 · 2 | Detailed of Solutions of CSE-Prelims:2018 • Paper-1 Analysis of 2018 Prelims: 1. The question paper was difficult as compared to the last year’s

Detailed of Solutions of CSE-Prelims:2018 • Paper-1 | 47

98

Which of the following has/have shrunk immensely/driedup in the recent past due to human activities ?

1. Aral Sea2. Black Sea3. Lake Baikal

Select the, correct answer using the code given below :(a) 1 only(b) 2 and 3 only(c) 2 only(d) 1 and 3 only

d Environmental Degradation

Aral sea and Lake Baikal have shrunkimmensely and dried up in the recentpast.

https://earthobservatory.nasa.gov/Features/WorldOfChange/aral_sea.php?src=fb

https://www.nationalgeographic.com/magazine/2018/03/drying-lakes-climate-change-global-warming-drought/

Difficult

Prelims Test Series-Test Number 18-QuestionNumber 2Q.2 The Aral Sea was recently in news as it hasreduced to 25 per cent of it’s size from the1960s. Between which of the following twocountries is it located?(a) Kazakhstan and Uzbekistan(b) Uzbekistan and Turkmenistan(c) Kazakhstan and Turkmenistan(d) Uzbekistan and Tajikistan

2. (a)The Aral Sea lake, has today reduced to 25 percent of it’s size from the 1960s. Since then, therivers that fed it, the Amu Darya and Syr Darya,have been re-routed to feed cotton production,and this has led to a shocking drop in its waterlevels. From a depth of 68 metres in the 1960s,today it stands at less than 10 metres. Lying in arain-shadow region, the rate of evaporation faroutpaces the rate of precipitation. It is locatedbetween Kazakhstan and Uzbekistan.

99

“Rule of Law Index” is released bywhich of the following ?(a) Amnesty International(b) International Court of Justice(c) The Office of UN Commissionerfor Human Rights(d) World Justice Project

dInternational

Institutions/Partnerships

World Justice Project: The WJP "Rule ofLaw Index" measures rule of lawadherence in 113 countries andjurisdictions worldwide.

https://worldjusticeproject.org/our-work/wjp-rule-law-index/wjp-rule-law-index-2017%E2%80%932018

Difficult

Magazine (Current affairs Made Easy)April 2018 edition (Page 41)

100

Which one of the following links all the ATMs in India ?

(a) Indian Banks’ Association(b) National Securities Depository Limited(c) National Payments Corporationof India(d) Reserve Bank of India

c Banking

National Financial switch under the NPCIlinks all ATMs in India. National FinancialSwitch (NFS) ATM network having 37members and connecting about 50,000ATMs was taken over by NPCI fromInstitute for Development and Researchin Banking Technology (IDRBT) onDecember 14, 2009. Over the span of fewyears, NFS ATM network has grownmany folds and is now the leadingmultilateral ATM network in the country.As on 31 st August’ 17, there were 941members that includes 101 Direct, 776Sub members, 56 RRBs and 8 WLAOsusing NFS network connected to morethan 2.37 Lac ATM.NFS has established a strong andsustainable operational model with in-house capabilities and today can becompared at par with other major and well-established switch networks. Theoperational functions and services are atpar with most of the global ATM networks.

https://www.npci.org.in/product-overview/national-financial-switch-product-overview

Medium